100

1500 Port & Mat

Embed Size (px)

Citation preview

Page 1: 1500 Port & Mat
Page 2: 1500 Port & Mat

APOSTILA DE EXERCÍCIOS E SIMULADOSAPOSTILA DE EXERCÍCIOS E SIMULADOSAPOSTILA DE EXERCÍCIOS E SIMULADOSAPOSTILA DE EXERCÍCIOS E SIMULADOSAPOSTILA DE EXERCÍCIOS E SIMULADOSAPOSTILA DE EXERCÍCIOS E SIMULADOSAPOSTILA DE EXERCÍCIOS E SIMULADOSAPOSTILA DE EXERCÍCIOS E SIMULADOSPORTUGUÊS & MATEMÁTICAPORTUGUÊS & MATEMÁTICAPORTUGUÊS & MATEMÁTICAPORTUGUÊS & MATEMÁTICAPORTUGUÊS & MATEMÁTICAPORTUGUÊS & MATEMÁTICAPORTUGUÊS & MATEMÁTICAPORTUGUÊS & MATEMÁTICA

20072007200720072007200720072007

A apostila foi elaborada com questões de vários concursos e tópicos não sequênciais, ou seja, estãomisturados para um melhor apredizado sem a famosa “decoreba” .

Bons estudos!

MATEMÁTICAMATEMÁTICAMATEMÁTICAMATEMÁTICAMATEMÁTICAMATEMÁTICAMATEMÁTICAMATEMÁTICA

1 Gastei 1/3 do meu dinheiro para pagamento de dívidas atrasadas, do que restou, coloquei 2/3 na poupança eainda fiquei com R$ 400,00. Qual era o valor correspondente a meu dinheiro?

Similar ao anterior.

• Salário = Z Aluguel = 3

1Z

Poupança = 3

2(Z -

3

1Z ) Resto = 400

Cálculo

Aluguel + poupança + resto = salário3

1Z+

3

2(Z -

3

1Z ) + 400= Z

3

1Z +

3

2Z -

9

2Z + 400= Z (

3

1Z +

3

2Z -

9

2Z + 400= Z)* 9

3Z + 6Z – 2Z + 9 * 400= 9Z 400*9 = 9Z –3Z - 6Z + 2Z

2Z= 400 * 9

Z= 2

9*400Z= 200 * 9

Z= 1800

A resposta é R$ 1.800,00

Page 3: 1500 Port & Mat

2- Decomponha os números:

Ex: 324= 3 centenas, 2 dezenas e 4 unidadesa) 421= _____________________________ b) 142= _______________________________c) 384=______________________________ d) 92=________________________________

3- Usando algarismos, complete corretamente:a) Naquele local havia, aproximadamente, uma centena de pessoas. Isto significa que havia,aproximadamente _____ pessoas no local.

b) Uma fábrica tem três centenas e quatro dezenas de empregados. Isto significa que a fábrica tem ______empregados.

FUVEST 2003Num bolão, sete amigos ganharam vinte e um milhõess, sessenta e três mil e quarenta e dois reais. Oprêmio foi dividido em sete partes iguais. Logo, o que cada um recebeu, em reais, foi:(a) 3.009.006,00(b) 3.009.006,50(c) 3.090.006,00(d) 3.090.006,50(e) 3.900.060,50

Resposta: (a)

1. Quantos anos têm Rogério, sabendo-se que o dobro da idade somado a sexta parte desta mesma idade, éigual a 26?

• 266

12 =+ ROGÉRIOROGÉRIO 6*)26

6

12( =+ ROGÉRIOROGÉRIO

• 6*2612 =+ ROGÉRIOROGÉRIO 6*2613 =ROGÉRIO

•13

6*26=ROGÉRIO

6*2=ROGÉRIO• ANOSROGÉRIO 12=

Page 4: 1500 Port & Mat

2. Numa festa filantrópica, o convite para homens custava R$ 15,00 e para mulheres, R$ 10,00 ( a graça deuma festa são as mulheres, não tem nem lógica se os convites custassem o mesmo preço para homens emulheres, as mulheres deveriam entrar de graça). Sabendo que o número de mulheres excede o número dehomens em 5 e que o valor arrecadado com os convites corresponde a R$ 550,00. Quantas mulheres forama festa?

•00,15$

00,10$

RHomens

RMulheres

=

=

5+= HomensMulheres

00,550$RoArrecadaçã =

• Sabemos que a arrecadação é a soma do dinheiro dos ingressos de homens e mulheres.• Vamos armar as relações demos: 550hom1510 =+ ensmulheres

• 5hom =− ensMulheres

•550155010

550hom15)5(hom10

=++

=++

HomnesHomens

ensens

50025

5055025

=

−=

Homens

Homens

•25

500=Homens 20=Homens

• Na festa havia 20 homens e 25 mulheres.

Escreva os numerais que estão entre 877 e o 883:________, ________, ________, ________, ________ 3) 8 dezenas e meia:_______O numeral ímpar, entre 864 e 868:____________ 5) 7 centenas e meia:____________

Usando os algarismos 5, 6 e 8 forme todos os números possíveis. ________, ________, ________, ________, ________, ________

Page 5: 1500 Port & Mat

3. Numa granja ha 870 aves, entre galinhas e frangos. Cada galinha abatida vale R$ 3,00 e o frangoabatido vale R$ 5,00. Considerando que o total apurado com o abate foi de R$ 3.150,00, quantos frangosforam abatidos?

•00,150.353

870

=+

=+

frangosgalinhas

frangosgalinhas

00,5$

00,3$

Rfrangos

Rgalinhas

=

=

• 150.353

)3(*)870(

=+

−=+

frangosgalinhas

frangosgalinhas

Multiplicando por (-3), facilita.

5402

150.353

261033

=

=+

−=−−

frangos

frangosgalinhas

frangosgalinhas

•2

540

5402

=

=

frangos

frangos

270=frangos

Distribua esses números no outro quadro, de forma que a soma deles em cada coluna seja 100

38 10 33 4

3 42 7 3

25 13 15 13

6 35 10 37

27 33 21 25

Page 6: 1500 Port & Mat

4. Num estacionamento há 76 veículos entre carros e motos. Sabendo que o total de rodas noestacionamento é de 212, pergunta-se, quantos carros e quantas motos há neste estacionamento?

212

2

4

76

=

=

=

=+

rodas

rodasmotos

rodascarros

motoscarros

Vamos armar o

sistema

21224

76

=+

=+

motoscarros

motoscarros

Vamos multiplicar por (-2) para facilitar

• 21224

)2(*)76(

=+

−=+

motoscarros

motoscarros

602

21224

15222

=

=+

−=−−

carros

motoscarros

motoscarros

2

60

302

=

=

carros

carros

46

30

=

=

motos

carros

Resolva as seguintes equações do 2º grau, sendo o conjunto U = R:

a) x2 + 7x = 0 S = {0, -7}

b) -3x2 + 9x = 0 S = {0, 3}

c) 2x2 + 3x = 0 S = {0, 3/2}

d) x2 + 9x = 0 S = {0,-9 }

e) y2 – 10 = 0 S = { }

f) 2x2 + 50 = 0 S = { }

g) -5r2 + 20 = 0 S = {-2, 2}

Page 7: 1500 Port & Mat

h) 9a2 = 25 S = {-5/3, 5/3}

i) (b + 6)(b – 4) = 2b + 12 S = {-6, 6}

j) 5y2- 9y – 2 = 0 S = {2, -1/3}

k) x2 – 9x + 20 = 0 S = {4, 5}

l) y2 + 9y + 14 = 0 S = {-2, -7}

m) b2 – 3b – 10 = 0 S = {-2, 3}

n) 2y2 + 7y + 6 = 0 S = {-2, -3/2}

o) 4y2 – 4y + 2 = 0 S = { }

p) 5t2 – 9t + 4 = 0 S = {1, 4/5}

q) 21m2 –26x + 8 + 0 S = {2/3, 4/7}

r) 4p2 – 20p + 25 = 0 S = {5/2}

s) x(x + 3) = 5x + 15 S = {-3, 5}

t) 2(a – 5) = a2 – 13 S = {-1, 3}

u) S = {-1/3, -1}

v) S = {2/3, -4/3}

w) x2 + 14x + 49 = 0 S = {-7}

x) 9y2 – 24y + 16 = 0 S = {4/3}

y) (3y + 2)(y – 1) = y(y + 2) S = {2, -1/2}

z) m2(m – 1) = m(m + 1)(m + 5) S = {0, -5/7}

Page 8: 1500 Port & Mat

1. Paulo possui em seu sítio 40 porcos, 8 vacas e 60 frangos. Determine a fração que representa osmamíferos. R = 5/3

2. Se 2/3 dos 40 alunos de uma sala usam óculos, calcule o número de alunos que não usam óculos. R =24

Se são decorridos 3/10 de um dia, que horas um relógio marcará neste momento? R =7h 20 min

3. Determine a soma dos termos de uma fração equivalente a 7/11 cujo numerador é 42. R = 108

4. Paulo gastou 5/7 do dinheiro que possuía em compras e lhe sobrou 400 reais. Determine a quantia quePaulo possuía antes da compara. R = 1000

5. Emerson comprou um moto, deu 2400 reais de entrada e o resto em 12 prestações iguais, cada qualcorrespondendo a 1/15 do preço da moto. Determine o preço pago pela moto. R = 12000

6. Numa certa cidade 3/16 dos moradores são de nacionalidade estrangeira. se total de habitantes é56400, calcule o número de habitantes brasileiros nessa cidade. R = 45825

7. O colégio Barão possui 2940 alunos. Sabendo-se que 3/10 desses alunos praticam futebol e 2/7praticam natação, determine o número de alunos que não praticam nenhuma das duas modalidades

esportivas. R = 1218.

8. Calcule a soma entre o dobro de 3/5 com o triplo de 16/9. R = 98/15

9. Nadia gastou 1/3 da farinha de trigo que possuía para fazer um bolo para suas amigas, mais tarderesolveu gastar 5/8 do restante da farinha para fazer uma torta. Determine a fração da farinha que

sobrará. R = 3/4

10. Determine a soma dos inversos dos números 10 e 4/10. R = 13/5

11. A professora de matemática de Aline pediu uma pesquisa informativa sobre os moradores do seubairro. Feita a pesquisa, Aline concluiu que: 1/2 dos moradores são menores de 18 anos e 1/2 dosrestantes são homens. Se as mulheres residentes nesse bairro são 130, determine o número de

moradores do bairro. R = 520

12. Nilson construiu sua casa em 3/7 do seu lote. Dias depois plantou frutas em 1/3 do restante.Determine que fração do terreno foi destinado ao plantio de frutas. R = 4/12

Page 9: 1500 Port & Mat

13. Aline querendo renovar seu material escolar, destinou 4/5 de sua mesada para compra destesmateriais. Logo após a compra, gastou 1/2 do que gastou em material escolar na compra de algumas

revistas. Determine a fração da mesada gasta na compra de livros. R= 2/5

1. Pedro propõe 16 problemas a um de seus amigos, informando que he dará 5 pontos por problemaresolvido e lhe tirará 3 pontos por problema não resolvido. No final, seu amigo tinha nota zero.

Quantos problemas seu amigo resolveu?

2. Um pai tem 30 anos a mais que seu filho. Se este tivesse nascido 2 anos mais cedo sua idade seria,atualmente, a terça parte da idade do pai. Calcule a idade atual do filho.

3. Um pai tem 37 anos e seu filho 7. Daqui a quantos anos, a idade do pai será o triplo da idade do filho?

4. Um menino tem 10 anos e seu pai 35 anos. Daqui a quantos anos a diferença das idades do pai e dofilho será 3/8 das sua soma.

5. Um feirante distribuiu laranjas entre três clientes, de modo que o primeiro recebe a metade daslaranjas, mais meia laranja; o segundo a metade das laranjas restantes, mais meia laranja e o terceiroa metade deste último resto, mais meia laranja. Sabendo-se que não sobrou nem uma laranja, calcule o

número total de laranjas e quantas foram dadas a cada cliente.

6. Dois estudantes juntos realizam uma tarefa em 5 horas. Sabendo-se que ficaram isolados, o primeirogasta a metade do tempo do segundo, calcule o tempo que o primeiro estudante gasta para realizar a

tarefa isoladamente.

7. Junior comprou uma calculadora por R$ 1.148,00 e a revendeu com lucro de 18% sobre o preço de venda.Qual o preço de venda.

8. Junior adquiriu uma mercadoria, obteve 5% de desconto sobre o preço de venda. Sabendo-se que elepagou R$ 19.000,00, calcule o preço de venda.

9. Num quintal há galinhas e coelhos num total de 8 cabeças e 22 pés. Quantas galinhas e quantoscoelhos existe no quintal?

10. Junior e Aline têm 100 livros. Se tirarem 25 livros de Junior e derem a Aline, ele ficarão com o mesmonúmero de livros. Quantos livros tem cada um?

11. Um reservatório, cuja capacidade é de 20 litros, é alimentado por uma torneira que fornece 3 litros deágua por hora. Calcule o tempo necessário para esvaziá-lo, retirando a água por uma torneira que sai

13 litro por hora.

Page 10: 1500 Port & Mat

RESPOSTASRESPOSTASRESPOSTASRESPOSTAS

1) 6 problemas 2) 12 anos 3) 8 anos 4) 10 anos e 10 meses 5) 1400 R$20.000,00 6) número de laranjas 7 cada cliente recebeu 4, 2 e 1 7) 7 horas e 30 minutos 8) 5 galinhas e 3 coelhos 9) 25 e 75 10) 2 horas

1.Resolva as seguintes equações do 1º grau, sendo U = Q:

a) 5x – 40 = 2 – xb) 20 + 6x = -2x + 26c) 3,5x + 1 = 3 + 3,1x d) 7p + 15 – 5p 10 = - 17 + 13p e) 13y – 5 = 11 + 9y f) 9t – 14 = 7t + 20 g) 5 – a – 11 = 4a – 22h) 2y + 21 – 6y = - 12 + y – 7 i) 3(x – 2) – (1 – x) = 13 j) 6(4 – t) – 55 = - 5(2t+ 3) l) 5 – 4(x – 1) = 4x – 3(4x – 1) – 4 m) 3(y – 3) + 4 = 2[-(y – 5) – 4(2y + 1)]

2. Em um terreiro há galinhas e coelhos, num total de 13 animais e 46 pés. Quantas galinhas e quantoscoelhos há nesse terreno?

3. A soma de dois números é 20. Se o dobro do maior é igual ao triplo do menor, determine o quadrado dadiferença desses dois números.

4. A soma da sexta parte com a quarta parte de um determinado número é o mesmo que a diferença entreesse número e 56. Qual é o número?

5. Uma empresa, em Viçosa, deu férias coletivas aos seus empregados. Sabe-se que 48% dos empregadosviajaram para o Rio de Janeiro, 28% viajaram para Belém e os 12 restantes ficaram em Viçosa. Nessascondições, quantos empregados tem essa empresa?

6. Uma casa, com 250 m2 de área construída, tem 4 dormitórios do mesmo tamanho. Qual é a área de cadadormitório, se as outras dependências da casa ocupam uma área de 170 m2?

Page 11: 1500 Port & Mat

7. Numa turma de 30 alunos, 6 escrevem com a mão direita e 2 escrevem com as duas mãos. Quantos alunosescrevem apenas com a mão direita?

8. Um reservatório contém combustível até 2/5 de sua capacidade total e necessita de 15 litros para atingir7/10 da mesma. Qual é a capacidade total desse reservatório?

9. A soma de três números inteiros e consecutivos é 60. Qual é o produto desses três números.

Uma senhora comprou 10 dúzias de ovos e 3 galinhas por R$ 1.500,00 ( que diabo de galinha cara é esta).Quanto custou cada ovo e cada galinha sabendo que uma galinha custa o mesmo que 10 ovos?

• Para resolver este problema, temos de desenvolver um sistema de equações do primeiro grau, onde umadas equações será referente ao preço e outra à igualdade entre ovos e galinhas.

• 15003120 =+ GALINHASOVOS OVOSGALINHA 101 =

• 150010*3120 =+ OVOSOVOS 150030120 =+ OVOSOVOS

• 1500150 =OVOS

150

1500=OVOS 10=OVOS

• 10010*10 ==GALINHA

Pedro, funcionário de uma empresa, recebeu o salário do mês e o gastou da seguinte maneira: 5

1 comprou

roupas; 20% do que sobrou, comprou alimentação. Sobraram R$ 160,00. Quanto Pedro gastou comalimentação?

• )5

1(*%20lim SSentaçãoA −= )250*

5

1250(*%20lim −=entaçãoa

• )50250(*5

1lim −=entaçãoA 200*

5

1lim =entaçãoA

• 00,40$lim RentaçãoA =

Page 12: 1500 Port & Mat

1. Sabendo-se que x + y + z = 18 e que, x/2 = y/3 = z/4, calcule x.

2. Três números são proporcionais a 1, 3 e 5. Calcule sua soma, sabendo-se que o seu produto é igual a960.

3. Humberto, Aline e Junior possuem uma livraria cujo o investimento foi de 9 mil reais. Humbertoentrou com 2 mil reais, Aline com 3 mil reais e Nilson com 4 mil reais. O lucro da livraria é divididoem partes proporcionais ao investimento de cada um deles. O lucro do mês de maio foi de 1800 reais,calcule quanto cada um vai receber neste mês.

4. Nilson vai dividir 360 mil reais entre seus três filhos, proporcionalmente ao número de membro dafamília de cada um deles. O primeiro tem esposa e 3 filhos, o segundo tem 2 filhos e é viúvo e o terceirotem esposa e 2 filhos. Quanto cada filho vai receber?

5. Será distribuído entre dois atletas o patrocínio de 42 mil reais, o melhor classificado receberá sua parteproporcional a 3 e o segundo, a 1. Determine quanto cada um recebeu.

6. Pedro quer dividir uma régua de 42 cm em parte proporcionais a 3, 5 e 6, quanto medirá cada parte.

7. A diretora de uma escola recebeu 372 livros para repartir proporcionalmente entre duas turmas. A 5ª Apossui 32 alunos e 5ª B possui 30 alunos. Quantos cadernos cada turma vai receber?

8. Divida 45 em partes inversamente proporcionais a 3, 4 e 6.

9. Divida 295 em partes inversamente proporcionais a 5, 1 e 9.

10. Divida 560 em partes inversamente proporcionais a 1, 3, 4 e 7.

GABARITOGABARITOGABARITOGABARITO

4 36

Humberto = 400, Aline = 600 e Nilson = 800 20, 15 e 10 45, 225 e 25

9408/29, 3136/29, 2352/29, 1344/29

Page 13: 1500 Port & Mat

5. A quantidade de selos que tenho, mais a sua metade, mais a quinta parte, mais sua terça parte menos200 somam um total de 410 selos. Quanto representa 30% de selos que possuo?

• Vários exercícios similares a este já foram solucionados, então veja somente o cálculo; • X= quantidade de selos

• 4102005

1

3

1

2

1=−+++ XXXX

• 30*)6105

1

3

1

2

1( =+++ XXXX

• 30*6106101530 =+++ XXXX•

• 300*6161 =X 61

300*61=X

• 300=X• A quantidade de selos do camarada é 300, mas ele está solicitando somente uma informação referente à

30% destes selos;

• 9030*310

300*3300*

10

3300*

100

30300*%30Re ======sposta

Resolva:

1.(2 x 3 - 4)2+10:5 R= 6 2.[16:8+(4:2+2 x 1)2]-5 R=13 3.(4 x 2-3x1)2+18:9 + 24 : 4 R= 33 4.21:7+(5 x 1-2x2)5+10 R= 14 5.[(5 + 12)-6]2+45:5+1 R= 11 6.20:4+6:3+(3 x 4-9 x 1)2 R= 16 7.[14+(4 x 5 - 3 x 6)3]-18:9 R= 20 8.(3 x 6 -7 x 2)3+ (16 : 8 – 12 : 12)6 R= 65 9.8+63:(14:7+6°)2+2 x 10 R= 35 10.8:4+(4-16:8)2+(10:5+45:9)° R= 7 11.[(64:8-19:19)2+50:25]-(3 x 5-7 x 2) R= 50 12.[54:9+(5 x 4+6 x 5-4 x 3)°-20:10 R=5 13.[10+6 X 4+(15:3+21:7)2]-5 x 10 R=48 14.12:6+16 : 8 +(48:6-55:11)2 – 2° R= 12 15.[10 x 2+3 x 5-(72:9-49:7)8+9° R= 35 16.[(3 x 4+5 x 4)-4 x 8]+19+(5 X 6 - 4 x 7)2 R= 5

Page 14: 1500 Port & Mat

1. Determine a soma de 0,018 km + 3421 dm + 0,054 hm, dando o resultado em metros.

2. O perímetro de um triângulo é 0,097 m e dois de seus lados medem 0,21 dm e 42 mm. Determine amedida do terceiro lado, em centímetros.

3. Uma mesa tem forma quadrada e seu perímetro é 480 cm. Calcule a área dessa mesa , em metrosquadrados.

4. Paulo comprou um sítio medindo 1,84 ha. Se cada metro quadrado custou 300 reais, quanto Paulopagou pelo sítio?

5. Resolva a expressão dando o resultado em metros cúbicos, 1425 dm3 + 0,036 dam3 +165000 cm3

6. Transforme:

Page 15: 1500 Port & Mat

a)3,621 dam3 para m3 b)16,4 m3 para dm3 c)314 cm3 para m3 d)0,01816 dm3 para cm3

7. O volume de um recipiente é 6500 cm3. Determine sua capacidade em litros.

8. Ana e Aline pesam juntas 78 kg. Se o peso de Ana é 42200g, qual será o peso de Aline?

9. José pagou por 2,5 toneladas de arroz a quantia de 3000 reais. Determine o preço pago por quilo dearroz.

10. Se 1kg de carne custa 3,25 reais, quanto pagarei por 3200 g?

11. Uma corrida de Formula 1 teve início às 2h 10min 42s. Se o vencedor faz um tempo de 3830s, a quehoras terminou a corrida?

12. Calcule o número de minutos que equivalem a 1mês 4dias 5horas

13. No bairro Nova Viçosa, durante o mês de novembro, choveu três vezes com as seguintes durações:25min 30s, 3h 42min 50s e 1h 34min 20s. Qual o tempo total de duração das chuvas neste bairrodurante o mês de novembro?

14. Para resolver 8 problemas Junior gasta 2h 48min 16s. Supondo que ele gasta tempos iguais em todasos problemas, qual é esse tempo?

RESPOSTASRESPOSTASRESPOSTASRESPOSTAS

365,5 m3,4 cm 1,44 m2

5 520 000reais

37,59 m3

6,5litros35800g1,20

10,403h 14min 32s49260 min5h 42min 40s

21min 2s

Page 16: 1500 Port & Mat

Uma torneira enche um tanque em 4 horas e outra em 6 horas. As duas torneiras ligadassimultaneamente, encherão o tanque em quanto tempo?

4

1G +

6

1G= 1 (

4

1G +

6

1G= 1 )*12 3G +

2G= 12 5G= 12

G= 5

12 horas G=

5

12* 60 minutos G= 144

minutos

G= 2 horas e 24 minutos

1. Se 15 operários levam 10 dias para completar um certo trabalho, quantos operários farão esse mesmotrabalho em 6 dias.

2. Com 100 kg de trigo podemos fabricar 65 kg de farinha. Quantos quilogramas de trigo são necessáriospara fabricar 162,5 kg de farinha?

3. Pedro comprou 2m de tecido para fazer uma calça. Quantos metros de tecido seriam necessários paraque Pedro pudesse fazer 7 calças iguais.

4. Num campeonato, há 48 pessoas e alimento suficiente para um mês. Retirando-se 16 pessoas paraquantos dias dará a quantidade de alimento?

5. Cinco pedreiros constróem uma casa em 300 dias. Quantos dias serão necessários para que 10pedreiros construam essa mesma casa?

6. Paulo trabalhou 30 dias e recebeu 15 000 reais. Quantos dias terá que trabalhar para receber 20 000reais?

7. Um carro com velocidade constante de 100 km/h, vai da cidade A até a cidade B em 3 horas. Quantotempo levaria esse mesmo carro para ir de A até B, se sua velocidade constante fosse 160 km/h?

8. O revestimento de um muro de 16 m de comprimento e 2,5 m de altura consome 84 kg de rebocopreparado. Quantos quilos de reboco serão necessários para revestir outro muro de 30 m decomprimento e 1,8 m de altura?

9. Mil quilos de ração alimentam 20 vacas durante 30 dias. Quantos quilos de ração são necessáriospara alimentar 30 vacas durante 60 dias?

Page 17: 1500 Port & Mat

10. Um livro tem 150 páginas. Cada página tem 36 linhas e cada linha, 50 letras. Se quisermos escrever omesmo texto em 250 páginas, quantas letras haverá em cada linha para que cada página tenha 30linhas?

11. Se 35 operários fazem uma casa em 24 dias, trabalhando 8 horas por dia, quantos operários serãonecessários para fazer a mesma obra em 14 dias trabalhando 10 horas por dias?

12. Três torneiras enchem uma piscina em 10 horas. Quantas torneiras seriam necessárias para encher amesma piscina em 2 horas?

13. Três operários constróem uma piscina em 10 dias. Quantos dias levarão 10 operários para construírema mesma piscina?

14. Duas máquinas empacotam 100 litros de leite por dia. Quantas máquinas são necessárias paraempacotarem 200 litros de leite em meio dia?

15. Numa laje de concreto de 6 cm de espessura foram gastos 30 sacos de cimento de 40 kg cada. Se a lajetivesse apenas 5 cm de espessura, quanto se gastaria de cimento.

RESPOSTASRESPOSTASRESPOSTASRESPOSTAS

11) 25 12) 250 kg 13) 14m 14) 45 dias 15) 150 dias

16) 40 dias 17) 1h 52 min 30 seg

2) 3000 kg 3) 36 linhas 4) 48 operários

5) 15 torneiras 6) 6 dias 7) 8 máquinas8) 100 Kg

Subtraindo-se 3/8 de um número, obtermos 60. Qual é o número?

O número é X

X - 8

3X= 60 (X -

8

3X= 60)*8 8X –

3X= 60*8 5X= 60*8

X= 5

8*60X= 12*8

X= 96O número é 96

Page 18: 1500 Port & Mat

Comprei uma moto por R$ 6000,00, dando de entrada uma quantia equivalente a um número cuja somaentre ele e seus 5/6 é R$ 2.200,00. Se o restante for pago em prestações mensais de R$ 200,00, quanto temposerá necessário para quitar o resto da dívida?

Preço da moto = 6000Preço da moto = 6000Preço da moto = 6000Preço da moto = 6000

Na entrada tem uma charadinha simples.X= entrada

X + 6

5X= 2200 Obs. Multiplique tudo pelo MMC.

(X + 6

5X= 2200)* 6 6X + 5X= 2200*6 11X= 2200*6

X= 11

6*2200X= 200*6 X= 1200

Agora que achamos o valor da entrada, podemos calcular o restante e dividir por 200 para ver em quantasparcelas vamos pagar.

Restante = 6000- entrada Restante = 6000- X Restante= 6000 - 1200

Restante = 4800 Quantidade de

Parcelas = 200

tanRe tes

Quantidade de parcelas = 200

4800Quantidade de

parcelas = 24

Page 19: 1500 Port & Mat

RESOLUÇÃO DA PROVA DE MATEMÁTICA DO BANCO DO BRASIL / 2003RESOLUÇÃO DA PROVA DE MATEMÁTICA DO BANCO DO BRASIL / 2003RESOLUÇÃO DA PROVA DE MATEMÁTICA DO BANCO DO BRASIL / 2003RESOLUÇÃO DA PROVA DE MATEMÁTICA DO BANCO DO BRASIL / 2003

Texto V – questões 13 e 14

Preparando-se para custear as despesas com a educação dos seus filhos, Carlos decidiu abrir uma poupançaprogramada para 120 meses duração, com rendimento mensal de 1%, em que os depósitos devem feitos noprimeiro dia de cada mês. O valor d(k), em reais, do depósito ser efetuado nessa poupança no k-ésimo mêsobedece às seguintes regras:

d(k) = 100, para k = 1, 2, ... , 12;

d(k + 12) – d(k) = 100, para k ³ 1.

Com base nas informações do texto V, julgue os itens abaixo.

1) d(42) = 400,00.

2) d(19) - d(15) = 0.

3) Durante o sétimo ano, o valor total a ser depositado por Carlos na poupança mencionada no texto ésuperior a R$ 8.500,00.

4) Se M(j) é o total a ser depositado por Carlos no ano j, poupança mencionada no texto, então os valores M(1), M(2), ..., M(10) formam, nessa ordem, uma progressão aritmética.

5) Para k1 = 3, se k1, k2, ..., k10 estão, nessa ordem, em progressão aritmética crescente de razão 13, então osvalores d(k1), d(k2), ..., d(k10) estão, nessa ordem, em uma progressão aritmética de razão 100

1) d(k) é uma função que representa o depósito a ser efetuado no K-ésimo mês,

d (42) representa, então, o depósito a ser efetuado no 42º mês

Para se achar d(42) tenho que utilizar os dados do problema, são eles:

a) d(k) = 100, para k = 1, 2, ... , 12, ou seja, d(1) = d(2) = d(3) = d(4) = d(5) = d(6) = d(7) = d(8) = d(9) = d(10) = d(11) = d(12) = 100

b) d(k + 12) – d(k) = 100, para k ³ 1.

Usando (b) tem-se:

d(42) = d(30+12) – d(30) = 100 (b.1)

d(30) = d(18+12) – d(18) = 100 (b.2)

Page 20: 1500 Port & Mat

d(18) = d(6+12) – d(6) = 100 (b.3)

em (b.3) tem-se d(18) = 100 + d(6) = 100+100 = 200

em (b.2) tem-se d(30) = 100 + d(18) = 100+200 = 300

em (b.1) tem-se d(42) = 100 + d(30) = 100 + 300 = 400

logo d(42) = 400 e o item é CORRETO.

2) Saber se a diferença dos depósitos no mês 19 e no mês 15 é nula, ou seja, d(19) - d(15) = 0.

d(19) = d(7+12) – d(7) = 100, logo d(19) = 100+d(7) = 100+100 = 200

d(15) = d(3+12) – d(3) = 100, logo d(15) = 100+d(3) = 100+100 = 200

logo: d(19) - d(15) = 200 – 200 = 0 e o item é CORRETO.

3) Os depósitos são feitos no primeiro dia de cada mês neste caso a cada ano há 12 depósitos. Durante osétimo ano teremos os depósitos d(73) a d(84).

Calculando o depósito no 73º mês d(73):

d(73) = d(59+12) – d(61) = 100 (3.1)

d(61) = d(49+12) – d(49) = 100 (3.2)

d(49) = d(37+12) – d(37) = 100 (3.3)

d(37) = d(25+12) – d(25) = 100 (3.4)

d(25) = d(13+12) – d(13) = 100 (3.5)

d(13) = d(1+12) – d(11) = 100 (3.6)

de (3.6) tem-se: d(13) = 100 + d(11) = 100 + 100 = 200

de (3.5) tem-se: d(25) = 100 + d(13) = 100 + 200 = 300

de (3.4) tem-se: d(37) = 100 + d(25) = 100 + 300 = 400

de (3.3) tem-se: d(49) = 100 + d(37) = 100 + 400 = 500

de (3.2) tem-se: d(61) = 100 + d(49) = 100 + 500 = 600

Page 21: 1500 Port & Mat

de (3.1) tem-se: d(73) = 100 + d(61) = 100 + 600 = 700

logo:

d(73) = d(74) = d(75) = d(76) = d(77) = d(78) = d(79) = d(80) = d(81) = d(82) = d(83) = d(84) = 700

Logo o valor total a ser depositado durante o sétimo ano vai ser de 12 depósitos de 700 no total de R$8.400,00 inferior a R$ 8.500,00.

A alternativa está ERRADA.

4) Dada uma nova função onde M(j) é o total a ser depositado por Carlos no ano j, logo M(1), M(2), ..., M(10) formam uma Progressão Aritmética (PA).

Lembrando que a Progressão Aritmética é uma seqüência numérica muito especial onde existe uma relaçãoentre todos os seus termos, a saber:

Termo anterior = termo posterior + razão, a razão é um valor constante.

Por exemplo:

2 4 6 8 10 é uma PA onde a razão é 2:

Termo posterior = termo anterior + 2

Repare:

4 – 2 = 6 – 4 = 8 – 6 = 10 – 8 = razão = 2

Analisando a função:

M(1) = total a ser depositado por Carlos no ano 1 = d(1) + d(2) + d(3) + d(4) + d(5) + d(6) + d(7) + d(8) +d(9) + d(10) + d(11) + d(12) = soma dos depósitos nos primeiros 12 meses = 12 . 100 = 1200

M(2) = total a ser depositado por Carlos no ano 2 = d(13) + d(14) + d(15) + d(16) + d(17) + d(18) + d(19) +d(20) + d(21) + d(22) + d(23) + d(24) = soma dos depósitos nos próximos 12 meses = 12 . 200 = 2400

Logo:

M(3) = 3600 M(4) = 4800 M(5) = 6000 M(6) = 7200 M(7) = 8400 M(8) = 9600 M(9) = 10800 M(10) =12000

Temos uma PA onde a razão é 1200, ou seja, o termo posterior = termo anterior – 1200.

Page 22: 1500 Port & Mat

A alternativa é CORRETA

5) Sendo k1 = 3, se, então os valores d(k1), d(k2), ..., d(k10) estão, nessa ordem, em uma progressão aritméticade

Se k1, k2, ..., k10 estão, nessa ordem, em progressão aritmética crescente de razão 13, tem-se:

k1 = 3 logo d(k1) = d(3) = 100

k2 = 3 + 13 = 16 logo d(k2) = d(16) = 200

k3= 16 + 13 = 29 logo d(k3) = d(29) = 300

k4= 29 + 13 = 42 logo d(k4) = d(42) = 400

k5 = 42 + 13 = 55 logo d(k5) = d(55) = 500

k6 = 55 + 13 = 68 logo d(k6) = d(68) = 600

k7 = 68 + 13 = 81 logo d(k7) = d(81) = 700

k8= 81 + 13 = 94 logo d(k8) = d(94) = 800

k9= 94 + 13 = 107 logo d(k9) = d(107) = 900

k10= 107 + 13 = 120 logo d(k10) = d(120) = 1000

Então os valores d(k1), d(k2), ..., d(k10) ou 100, 200, 300, 400, 500, 600, 700, 800, 900, 1000, estão, nessaordem, em uma progressão aritmética de razão igual a 100.

Alternativa CORRETA

Gabarito Final

CCECC

Page 23: 1500 Port & Mat

Resolver as equações1. 2x + 4 = 102. 5k - 12 = 203. 2y + 15 - y = 224. 9h - 2 = 16 + 2h

Determine a solução do sistema:

x + y = 2x - y = 0

Determinar o valor de X para que a razão X/3 esteja em proporção com 4/6.

Solução: Deve-se montar a proporção da seguinte forma:

xxxx

3333

====

4444

6666Para obter X=2.

Exercícios1.Calcular o discriminante de cada equação e analisar as raízes em cada caso:1.x² + 9 x + 8 = 02.9 x² - 24 x + 16 = 03.x² - 2 x + 4 = 04.3 x² - 15 x + 12 = 05.10 x² + 72 x - 64 = 02.

Resolver as equações:1.x² + 6 x + 9 = 02.3 x² - x + 3 = 03.

Page 24: 1500 Port & Mat

2 x² - 2 x - 12 = 04.3 x² - 10 x + 3 = 0Resolver as equações do segundo grau fracionárias:1.x + 6/x = -72.(x+2)/(x+1) = 2x/(x-4)3.(2-x)/x + 1/x² = 3/x4.(x+2)/(x-2) + (x-2)/(x+2) = 1

01 – Num depósito existem 24 extintores de incêndio, sendo de espuma química e dióxido de carbono.Sabendo-se queo de dióxido de carbono é o triplo do de espuma química, conclui-se que o número de extintores de espumaquímicaexistentes nesse depósito é:a) 3 b) 4 c) 5 d) 6

RESOLUÇÃO:Vamos observar que é melhor adotar as iniciais das palavras. Pois se adotarmos x e y fica um pouco confusonahora de dar a resposta.E = número de extintores de espuma químicaD = número de extintores de dióxido de carbonoE + D = 24 E + D = 24D = 3E - 3E + D = 0

Como queremos o valor de E, basta multiplicar a segunda equação por (-1) e com o método da adiçãoencontraremos o valor de E.E + D = 24 E + D = 24-3E + D = 0 3E - D = 04E = 24E = 24/4E = 6

O número de extintores de espuma química é de 6 extintores.Opção: D

Page 25: 1500 Port & Mat

02 – Eu tenho o dobro da idade da minha filha. Se a diferença de nossas idades é 23 anos, minha idade é:a) 40 anos b) 46 anos c) 48 anos d) 50 anos

RESOLUÇÃO:M = minha idadeF = idade da filhaM = 2F M – 2F = 0 M – 2F = 0M – F = 23 M – F = 23 . ( - 2 ) - 2M + 2F = - 46- M = - 46 . (-1)M = 46

A minha idade é 46 anos.Opção: B

03 – A soma da minha idade com a da minha filha é 72. Daqui a 3anos a minha idade será o dobro da idade daminha filha. A minha idade atual , em anos é:a) 47 b) 49 c) 51 d) 53

RESOLUÇÃO:M = minha idadeF = idade da filhaM + F = 72 M + F = 72 M + F = 72M + 3 = 2.(F + 3) M + 3 = 2F + 6 M - 2F = 6 - 3M + F = 72 . ( 2 ) 2M + 2F = 144M – 2F = 3 M – 2F = 33M = 147M = 147/3M = 49

A minha idade é 49 anos.Opção: B

Calcule:

1.m.m.c (3, 4, 6) 2.m.m.c (2, 4, 8) 3.m.m.c (3, 6, 9) 4.m.m.c (4, 8, 10) 5.m.m.c (6, 12, 15) 6.m.m.c (6, 15, 18) 7.m.m.c (8, 12, 20) 8.m.m.c (9, 15, 27) 9.m.m.c (12, 16, 24)

Page 26: 1500 Port & Mat

10.m.m.c (12, 15, 21) 11.m.m.c (20, 25, 40) 12.m.m.c (16, 32, 48) 13.m.m.c (12, 32, 48) 14.m.m.c (15, 25, 40) 15.m.m.c (24, 30, 45) 16.m.m.c (25, 50, 75) 17.m.m.c (32, 48, 64) 18.m.m.c (30, 45, 60) 19.m.m.c (6, 12, 18, 30) 20.m.m.c (35, 50, 70, 100)

21.Dois carros partem juntos, a fim de dar voltas em torno de uma pista de corrida. O carro mais rápidodemora 3 minutos para completar uma volta e o outro carro demora 5 minutos. Após quanto tempo os carrosirão se encontrar novamente? 35

RESPOSTASRESPOSTASRESPOSTASRESPOSTAS

1) 12 2) 8 3) 18 4) 40 5) 60 6) 90 7) 120 8) 135 9) 420 10) 20011) 96 12) 60 13) 600 14) 150 15) 192 16) 180 17) 180 18) 700

Calcule:

1. m.d.c (16, 18 20)

2. m.d.c (15, 20, 30)

3. m.d.c (14, 21, 28)

Page 27: 1500 Port & Mat

4. m.d.c (14, 28, 35)

5. m.d.c (35, 45, 50)

6. m.d.c (24, 30, 32)

7. m.d.c (50, 60, 80)

8. m.d.c (56,64,72)

9. m.d.c (56,66,76)

10. m.d.c (100,108,120)

11. m.d.c (125,250,300)

12. m.d.c (128,256,512)

13. m.d.c (81,243,729)

14. m.d.c (250,350,400)

15. m.d.c (24,48,96,144)

16. m.d.c (25,75,150,300)

17. m.d.c (20,40,60,80)

18. m.d.c (36,72,84,108)

19. m.d.c (18,36,48,96)

20. m.d.c (28,56,70,140)

Reaspostas

R = 2R = 5R = 7R = 2R = 5R = 2R = 10

Page 28: 1500 Port & Mat

R = 8R = 2R = 4R = 25R = 128R = 81R = 50R = 24R = 25R = 20R = 12R = 6R =14

1. Determine a porcentagem pedida em casa caso.

a) 25% de 200 b) 15% de 150 c) 50% de 1200 d) 38% de 389 e) 12% de 275 f) 11,5% de 250 g) 75% de 345 h) 124% de 450

2. Se 35 % dos 40 alunos da 5ª série de um colégio são homens, quanto são as mulheres?

3. Aline foi comprar uma blusa que custava R$ 32,90, e conseguiu um desconto de 12%. Quantos Aline pagoupela blusa?

4. Nilson decidiu compra um sítio e vai dar como entrada 25% do preço total, que corresponde a R$ 25000,00. Qual o preço do sítio.

5. Ricardo comprou um terreno e, por ter pagado à vista, ganhou 15% de desconto, fazendo uma economia deR$ 2 250,00. Determine o preço deste terreno que Ricardo vai comprar.

6. Paulo recebeu a noticia de que o aluguel da casa onde mora vai passar de 154 reais para 215,60 reais. Dequanto será o percentual de aumento que o aluguel vai sofre.

7. Na cidade de Coimbra 6% dos habitantes são analfabetos. Os habitantes que sabem ler são 14 100

Page 29: 1500 Port & Mat

pessoas. Quantos indivíduos moram nesta cidade?

8. Nádia teve um reajuste salarial de 41%, passando a ganhar R$ 4 089,00. Qual era o salário antes doreajuste?

9. Em certo trimestre as cadernetas de poupança renderam 2,1% de correção monetária. Paulo deixou R$1000,00 depositados durante três meses. Quanto tinha no fim do trimestre.

10. Em um colégio 38% dos alunos são meninos e as meninas são 155. Quantos alunos têm esse colégio?

RESPOSTASRESPOSTASRESPOSTASRESPOSTAS

16. a) 50b) 22,50c) 600d) 147,82e) 33f) 28,75g) 258,75h) 5517. 26 18. 28,95 19. 100 000 20. 15 000 reais 21. 40% 22. 15 000 reais 23. 2 900 reais 24. 1 021 reais 25. 210 alunos

1. Determine o número de tacos de 6cm de largura por 24cm de comprimento necessários para assoalharuma sala de 3,6m de largura por 4,2cm de comprimento.

2. Uma caixa d'água comporta 360 litros e tem uma torneira que a enche em 15 horas e outra que aesvazia em 20 horas. Abrindo-se as duas torneiras simultaneamente, qual o número de horasnecessárias para encher a caixa?

3. Um pátio retangular tem 1,8dam de comprimento e 75dm de largura. Para pavimentar o pátio foramescolhidos ladrilhos quadrados de 25cm de lado. Determine o número de ladrilhos gastos.

4. Determine o número de voltas que uma roda de 50dm de raio precisa dar, para percorrer uma distânciade 628km.

5. Uma lavoura de grãos com 100km2 de área plantada fornece uma produção de 5 toneladas por hectare.Sabendo-se as máquinas usadas colheram 2000 toneladas por dia. Qual o tempo gasto para se fazer acolheita desta lavoura?

Page 30: 1500 Port & Mat

6. Um trem, com velocidade de 48km/h, gasta 1 hora e 20 minutos para percorrer certa distância. Parafazer o mesmo percurso a 60km/h o trem gastaria

7. Uma turma de operários faz uma obra, cujo coeficiente de dificuldade é 0,2 em 8 dias. Em quantosdias a mesma turma faria outro trabalho, com coeficiente de dificuldade 0,25?

8. Para fazer um determinado serviço, 15 homens gastam 40 dias; para fazer o mesmo serviço em 30 diasquantos novos operários têm de ser contratados

9. Numa viagem de automóvel, uma pessoa gastou 9 horas andando à velocidade de 80km/h. Na volta,

quanto tempo irá gastar, se andar com velocidade de 100km/h?

10. As dimensões de um tanque retangular são 1,5m, 2,0m e 3,0m. Com uma torneira de vazão10litros porminuto, qual o menor tempo gasto para enchê-lo?

11. Se a massa de 1000cm3 de certo líquido é 3,75kg, qual a massa de 1,35m3 do mesmo líquido?

12. Trabalhando 10 horas por dia, certa máquina faz um trabalho em 240 dias. Se a mesma máquinafuncionar 8 horas por dia, em quanto dias fará o mesmo trabalho?

13. Um edifício projeta uma sombra de 12m no mesmo instante em que um objeto de 2m de altura projetauma sobra de 80cm. Calcule a altura do edifício

14. Uma torneira enche um tanque de 100 litros em 1 hora, enquanto uma segunda gasta 2 horas. As duasjuntas encherão o tanque em quanto tempo?

15. Para vender todos os ingressos de um cinema Aline gasta 15 minutos e Junior 30 minutos.Trabalhando juntos, qual o tempo gasto para venderem os ingressos?

16. Para escrever um texto, usando 54 letras por linha, foram necessárias 15 linhas. Quantas linhas serãonecessárias para 30 letras em cada linha?

17. Para fazer uma cerca, são necessários 80 postes distantes entre si de 2,5m. Quantos postes serãonecessários, se a distância entre eles for de 2m?

18. Uma vara de 5 m, colocada em posição vertical, projeta no chão uma sombra de 3,5m. Calcule a alturade um prédio que, na mesma hora e o mesmo local, projeta uma sombra de 12,6m.

19. Com 72kg de lã, faz-se uma peça de fazenda de 63m de comprimento. Quantos kg de lã seriamnecessários para fazer 84m da mesma fazenda?

20. Numa cidade, há 22410 estrangeiros. A razão entre o número de habitantes é de 18 para 100. Quantoshabitantes há na cidade?

Page 31: 1500 Port & Mat

RESPOSTASRESPOSTASRESPOSTASRESPOSTAS

18) 1050 19) 60 20) 2160 21) 20.000 voltas 22) 25 dias. 23) 1h 4min 24) 10 dias?

25) 5 26) 7h 12min 27) 15 h 28) 5062,5kg 29) 300. 30) 30m 31) 40min

32) 10min 33) 27 34) 100 35) 18m 36) 96 37) 124.500

1. Se (x; y) e (y; 12) são sucessões de números diretamente proporcionais, então:

a) x = y/2 b) x = y/3c) x = y/4d) x = y/5e) y = 3x/12

2. Um fazendeiro tem ração para alimentar 50 galinhas durante 80 dias. Decorridos 15 dias resolveuvender 10 galinhas. De quanto poderá ser aumentada a ração diária de cada galinha durante o resto doperíodo?

a) 5/4b) 3/5c) 1/5d) 1/4e) 3/4

3. O salário de uma pessoa era, em setembro de 1998, R$ 12.000,00 e em dezembro de 1998, R$13.886,46. Sabe-se que as taxas de reajustes aplicadas ao seu salário em outubro e novembro foramrespectivamente de 5% e 3%. Qual foi a taxa de reajuste relativa ao mês de dezembro?

a) 7% b) 8% c) 10% d) 9% e) 11%

4. Um capital de R$ 10.000,00 foi aplicado a juros compostos de 120% a.a., com capitalização mensal,por 4 meses. Determine a taxa de juros quadrimestral em que o mesmo poderia ser aplicado a fim dese obter o mesmo juro.

a) 10% aqb) 33,1% aqc) 40% aqd) 41,46% aqe) 46,41% aq

Page 32: 1500 Port & Mat

5. Uma duplicata no valor de R$ 1.440,00 foi descontada por dentro 5 meses antes do vencimento àtaxa simples de 48% a.a. O valor líquido dessa duplicata foi de:

a) R$ 1.000,00b) R$ 1.200,00c) R$ 1.260,00d) R$ 1.340,00e) R$ 1.400,00

6. Um hóspede de um hotel teve que pagar R$ 174,00 por quatro dias de hospedagem. Pela estada deoito dias outro hóspede pagou R$ 342,00 num quarto do mesmo tipo. Sabe-se que a conta de cada umdos hóspedes foi calculada multiplicando-se o valor da diária pelo número de dias de permanência eadicionando-se ao resultado uma taxa fixa de hospedagem. Nestas condições, considere asafirmativas abaixo:

I – No cálculo feito a despesa é uma função linear do número de dias de permanência.I I – A taxa fixa que foi cobrada de cada hospede foi de R$ 4,00.I I I – Por uma estada de 5 dias nas mesmas condições a conta do hotel seria de R$ 216,00.

Assinale a única alternativa correta:

a) Somente a afirmativa I está correta.b) Somente a afirmativa I I está correta.c) Somente as afirmativas I e I I estão corretas.d) Somente as afirmativas I e I I I estão corretas. e) Somente as afirmativas I I e I I I estão corretas.

7. Para fazer um cercadinho para uma horta no quintal de casa, o dono da casa dispõe de 16 metros detela de arame. Se ele aproveitar o muro que fica no fundo do quintal como um dos lados do cercadinho,ele poderá fazer o cercadinho com um formato retangular usando os 16 metros de tela para formar osoutros três lados do retângulo. Nestas condições, julgue as afirmativas abaixo:

I – A área que o cercadinho terá não depende das medidas dos lados do retângulo formado pois eleusará sempre os mesmos 16 metros de tela.I I – A área que o cercadinho terá depende das medidas escolhidas para os lados do retângulo formadoe pode ser expressa como uma função quadrática da medida de um dos lados do retângulo.I I I – A maior área possível do cercadinho será obtida quando o maior lado do retângulo formado tiver12 metros.

Assinale a única alternativa correta:

a) Somente a afirmativa I está incorreta. b) Somente a afirmativa I I está incorreta.c) Somente as afirmativas I e I I estão incorretas.d) Somente as afirmativas I e I I I estão corretas.e) Somente as afirmativas I I e I I I estão incorretas.

Page 33: 1500 Port & Mat

8. Se o número 225 for dividido em 3 partes, formando uma progressão aritmética, de maneira que aterceira parte exceda a primeira em 140 unidades, essas três partes serão:

a) números primos entre si.b) todas múltiplas de 3 e de 5.c) todas menores que 100.d) todas maiores que 10.e) todas fatores do número 54.375.

9. Um triângulo isósceles tem 32 cm de perímetro e 8 cm de altura em relação à base (isto é, comrelação ao lado diferente dos demais). A área deste triângulo, em centímetros quadrados, é:

a) 24b) 16c) 96d) 100e) 48

10. Uma piscina infantil, dessas infláveis, tem fundo circular com 2 metros de diâmetro e tem 40centímetros de altura. Para enchê-la com água até três quartos de sua altura, o número aproximado delitros necessário será:

a) 924b) 942 c) 1.265d) 1.256e) 1.526

GABARITOGABARITOGABARITOGABARITO01. C 06.D02. D 07. A03. A 08. E04.E 09.E05. B 10. B

Num depósito existem 24 extintores de incêndio, sendo de espuma química e dióxido de carbono. Sabendo-se que o de dióxido de carbono é o triplo do de espuma química, conclui-se que o número de extintores deespuma química existentes nesse depósito é:a) 3 b) 4 c) 5 d) 6

Page 34: 1500 Port & Mat

RESOLUÇÃO:Vamos observar que é melhor adotar as iniciais das palavras. Pois se adotarmos x e y fica um pouco confusona hora de dar a resposta.E = número de extintores de espuma químicaD = número de extintores de dióxido de carbonoE + D = 24 E + D = 24D = 3E - 3E + D = 0

Como queremos o valor de E, basta multiplicar a segunda equação por (-1) e com o método da adiçãoencontraremos o valor de E.E + D = 24 E + D = 24-3E + D = 0 3E - D = 04E = 24E = 24/4E = 6

O número de extintores de espuma química é de 6 extintores.Opção: D

Page 35: 1500 Port & Mat

PORTUGUÊSPORTUGUÊSPORTUGUÊSPORTUGUÊSPORTUGUÊSPORTUGUÊSPORTUGUÊSPORTUGUÊS

(AFC/SFC 2002) No texto abaixo, foram introduzidos erros. Para saná-los, foram propostas algumassubstituições.

Julgue as substituições e depois assinale a opção que contém a seqüência das alterações necessárias paraadequar otexto ao padrão culto formal do idioma.“O conceito de tributo, face sua interpretação nos conformes à Constituição, tem essa peculiaridade: deveobedecer ao princípio da legalidade estrita. Cumpre ressaltar mais uma vez: não há possibilidade dediscricionariedade na definição legislativa do tributo, mas só teremos tributo se o dever de pagar umaimportância ao Estado for vinculado à previsão de ter riqueza.”(Nina T. D. Rodrigues, com adaptações)IV. substituir “discricionariedade”(l .5 e 6) por “discricionaridade”

Item ERRADO.

Comentário.

Se houvesse dúvida com relação à grafia, o candidato poderia buscar uma outra palavra parecida (ou seja, umparadigma) que tivesse passado pelo mesmo processo: SÉRIO-> SERIEDADE SOLIDÁRIO ->SOLIDARIEDADE SÓCIO -> SOCIEDADE SÓBRIO -> SOBRIEDADEDISCRICIONÁRIO -> DISCRICIONARIEDADE

Como é bem maior a quantidade de vocábulos terminados por IO, em comparação com os de terminaçãoEO, pode ocorrer a “contaminação” e, por conseguinte, erro na grafia de vocábulos como HOMOGÊNEO(confira como foi a questão 17 da prova para AFRF 2003, mais adiante). Então,para dissipar dúvidas, vamosbuscar um paradigma. O que acontece com essa palavra é o mesmo que ocorre com:CORPÓREO -> CORPOREIDADEIDÔNEO -> IDONEIDADECONTEMPORÂNEO -> CONTEMPORANEIDADEINSTANTÂNEO -> INSTANTANEIDADEESPONTÂNEO -> ESPONTANEIDADE Dos exemplos acima, é claro que o candidato inteligente vai buscar como paradigma o segundo, não é? Ouaté outro mais comum ainda, pense aí você... Essa dica do paradigma vai também ajudar – e muito – emrelação a conjugação verbal. Mas isso fica para umpróximo ponto.

(Fiscal do Trabalho/1998) Leia o texto seguinte para responder às questões.Existe atualmente, uma série de leis que protegem o trabalhador. O trabalhador cede o seu trabalho e quem

Page 36: 1500 Port & Mat

toma o trabalho do operário vê-se obrigado a cumprir um conjunto de obrigações, que estão agrupadas na leidenominada Consolidação das Leis do Trabalho (CLT).Sempre que houver a despedida de um trabalhador motivada ou imotivadamente, e qualquer uma das partes,principalmente o empregado, considerar que, aquilo que foi tratado por ocasião de seu contrato de trabalhonão foi cumprido na sua demissão, ele vai ao seu sindicato e argüi perante as Juntas de Conciliação eJulgamento, os seus direitos. Quando é argüida a insalubridade ou a pericolosidade, o juíz requisitaráperícias a cargo do médico dotrabalho ou engenheiro do trabalho.(Baseado em João Alberto Maeso Montes e Dirceu Francisco Araújo Rodrigues)Com relação à ortografia, ocorre(m) no textoa) nenhum errob) dois errosc) quatro errosd) três errose) um erro

GABARITO: D

(Analista Com.Exterior/1998) Leia o texto seguinte para responder às questões.Em Direito Tributário, a expressão sanções políticas corresponde a restrições ou proibições impostas aocontribuinte,como forma indireta de obrigá-lo ao pagamento do tributo, tais como a interdição do 5estabelecimento, a apreensãode mercadorias, o regime especial de fiscalização, entre outras. Qualquer queseja a restrição que implique cerceamento da liberdade de exercer atividade lícita é inconstitucional, porquecontraria o disposto nos artigos 5o, inciso XIII, e 170, 10 parágrafo único, do estatuto maior do país.O Supremo Tribunal Federal sumulou sua jurisprudência no sentido de serem inconstitucionais as sançõespolíticas. A Súmula 70 diz que é inadimissível a interdição de estabelecimento como meio coercitivo 15 paracobrança de tributo. Diz a Súmula 323 que é inaceitável a apreensão de mercadorias como meio coercitivopara pagamento de tributo, e a 547, estabelece que não é lícito à autoridade proibir que o contribuinte emdébito adquira estampílias, despache 20 mercadorias nas alfândegas e exerça suas atividades profissionais.Não obstante inconstitucionais, as sanções políticas, que no Brasil remontam aos tempos da ditadura deVargas, vêm-se tornando, a cada dia, mais numerosas e 25 arbitrárias, consubstanciando as mais diversasformas de restrições a direitos do contribuinte, como forma oblíqua de obrigá-lo ao pagamento de tributos ou,às vezes, como forma de retaliação contra o contribuinte que vai a juízo pedir proteção contra cobrançasilegais.(Baseado em Hugo de Brito Machado, Direito e Justiça, CB,27/04/1998)Em relação à ortografia, ocorre(m) no textoa) um errob) dois errosc) nenhum errod) três errose) quatro erros

Gabarito: B.

Page 37: 1500 Port & Mat

EXERCÍCIOS DE CONCURSOS

01) EMM - Há predicado verbo-nominal em:a) Ela descansava em casa.b) Todos cumpriram o juramento.c) Ele vinha preocupado.d) Ele está abatido.e) Ela marchava alegremente.

02) EMM - Em “Sacou da arma”, a função sintática do termo sublinhado é:a) objeto direto preposicionado;b) objeto indireto;c) adjunto adverbial de meio;d) objeto direto;e) complemento nominal.

03) Colégio Naval - “(...) o guri curioso que eu era (...)”O termo sublinhado, na passagem acima, apresenta a função sintática de:a) sujeito;b) objeto direto;c) pronome relativo;d) predicativo do sujeito;e) adjunto adverbial de intensidade.

04) EPCAR - Em relação à palavra “se”, utilize a chave a seguir para classificá-la corretamente e aponte aresposta encontrada.1) Pronome Apassivador2) Objeto Indireto3) Objeto Direto4) Conjunção subordinativa condicional5) Conjunção subordinativa integrante6) Índice de indeterminação do Sujeito

( ) Os bandidos deram-se as costas e atiraram.( ) Só trabalho se me pagarem bem.( ) Nesta rua dorme-se tranqüilo.( ) Nada se fez para encontrá-lo.( ) As irmãs se abraçaram felizes.( ) Veja, por favor, se ela já chegou.

a) 2 - 4 - 1 - 5 - 6 - 3;b) 3 - 4 - 6 - 1 - 2 - 5;

Page 38: 1500 Port & Mat

c) 2 - 4 - 6 - 1 - 3 - 5;d) 3 - 2 - 6 - 4 - 5 - 1;e) 6 - 2 - 4 - 5 - 1 - 3.

05) ESPCEX - Quantos são os objetos diretos dos três períodos juntos?a) “Rubião esqueceu a sala, esqueceu a mulher, esqueceu até a si.”b) Não devias consentir em tais loucuras nem comer do mesmo pão.c) Faleceu João, a quem muito estimávamos.Resposta: a) __________ b) __________ c) __________

06) ESPCEX - Assinale as afirmações verdadeiras sobre a oração “Precisa-se de muitos operáriosespecializados.”1 Oração sem sujeito.2 Oração de sujeito indeterminado.4 Oração absoluta.8 Oração de verbo intransitivo direto.16 O se é pronome apassivador.32 O se é índice de indeterminação do sujeito.64 O predicado da oração é verbal.

Na frente de cada linha existe um número (dentro de um círculo).Some os números correspondente às linhas nas quais você assinalou afirmações verdadeiras.Resposta: __________

07) Colégio Naval - Assinale a oração que não possui sujeito:a) A noite caiu repentinamente sobre a cidade.b) Nesse mês, vai fazer um ano da sua partida.c) Choveram tomates sobre o orador.d) O dia amanheceu bastante límpido.e) Não havia existido ninguém com tantas qualidades.

08) Escola Naval - Observe a seguinte frase atribuída a José de Alencar: “Ficaram-lhe as mãosensangüentadas.” -A palavra grifada terá a seguinte função sintática:a) objeto indireto;b) complemento nominal;c) predicativo do sujeito;d) predicativo do objeto direto;e) adjunto adnominal.

09) Colégio Naval - Perdoai esta logomaquia; o estilo ressente-se da exaltação da minha alma. Achei umhomem. Se aquele cínico Diógenes pode ouvir, do lugar onde está, as vozes cá de cima, deve cobrir-se de

Page 39: 1500 Port & Mat

vergonha e tristeza;achei um homem.Em “(...) deve cobrir-se de vergonha”, podemos afirmar que o sujeito:a) É “aquele cínico Diógenes” (simples)b) É indeterminado;c) É “vergonha e tristeza” (composto e posposto ao verbo);d) Trata-se de uma oração sem sujeito;e) Está na oração principal do período.

10) EMM - Na oração “Ele se julga feliz”, a função sintática dos termos sublinhados é:a) Objeto direto - adjunto adverbial de modo.b) Objeto indireto - objeto direto.c) Pronome reflexivo - predicativo.d) Objeto direto - predicativo.e) Sem função - objeto direto.

11) EMM - A única oração com sujeito simples é:a) Existem algumas dúvidas.b) Compraram-se livros e cadernos.c) Precisa-se de ajuda.d) Faz muito frio.e) Há alguns problemas.

12) Colégio Naval - Assinale a opção em que o termo grifado seja objeto direto interno ou cognato:a) “(...) sonhei coisas (...)”b) “(...) apenas ia vivendo devagar a vida lenta dos mares do Brasil”c) “(...) pois ainda há um Brasil bom (...)”d) ‘(...) que poderia trocar por roscas amanteigadas (...)”e) “(...) é capaz de haver mulher também (...)”

13) ITA“Foi solto no gramado e a tela fina de arame é escarmento ao rei dos animais. Não mais que um caco de leão(...)”“(...) sobre o focinho contei nove ou dez moscas, que ele não tinha ânimo de espantar.”“Um dos presentes explica que o bicho tem as pernas entrevadas, (...)”“Um de nós protesta que deviam servir-lhe a carne em pedacinhos.”

Dos conectivos grifados nos fragmentos acima, somente um acumula em si os papéis de ligar orações edesempenhar uma função sintática (núcleo) na estrutura da oração introduzida.Assinale a opção que o contiver:a) “(...) e a tela fina de arame é escarmento ao rei dos animais.”b) “(...) não mais que um caco de leão.”c) “(...) que ele não tinha ânimo de espantar.”d) “(...) que o bicho tem as pernas entrevadas,”

Page 40: 1500 Port & Mat

e) “(...) que deviam servir-lhe a carne em pedacinhos.”

14) ESPCEX - No período:“A alma do carpinteiro não pode ser mais bruta que a madeira.”As funções dos termos grifados são, pela ordem:a) predicativo do sujeito - sujeito;b) predicativo do sujeito - objeto direto;c) adjunto adverbial - sujeito;d) objeto direto - predicativo do sujeito.

15) EPCAR - O termo sublinhado exerce a função de objeto indireto, exceto em:a) Lembrei-lhe a data de aniversário de sua mãe.b) Perdi a cabeça durante a discussão e dei-lhe na cara.c) Devido a problemas de saúde, proibiram-lhe que fumasse.d) Incumbiram-lhe que entregasse a encomenda.e) Com certeza, pagou-lhe com bastante atraso.

16) ESPCEX - Na oração:“De que, mulher, você ainda tem dúvida?”,As funções dos termos grifados são, pela ordem:a) complemento nominal - aposto - objeto direto;b) objeto indireto - sujeito - objeto direto;c) objeto indireto - vocativo - predicativo do sujeito;d) complemento nominal - vocativo - objeto direto.

17) Magistério Estadual - RJ“a valer tanto quanto a primeira natureza que, no indivíduo, em não mais havendo, não se sabe como oraseria”“e isso parece claramente espelhar-se na forma por que, na modernidade, a linguagem oficial se vemalterando”As orações subordinadas em destaque exercem, respectivamente, as funções sintáticas de:a) sujeito / adjunto adnominal;b) sujeito / adjunto adverbial de causa;c) objeto direto / adjunto adverbial de causa;d) adjunto adverbial de modo / adjunto adnominal;e) adjunto adverbial de modo / adjunto adverbial de causa.

18) ESPCEX - Uma oração de predicado verbo-nominal está exemplificada em:a) Nesta casa, só se aceitam pessoas educadas.b) Os viajantes chegaram cedo ao destino.c) Estava irritado com as brincadeiras.d) Compareceram todos atrasados à reunião.

Page 41: 1500 Port & Mat

19) ESPCEX - “(...) Se algum dia fosse poeta e quisesse compor um poema nacional, pediria a Deus que mefizesse esquecer por um momento as minhas idéias de homem civilizado. Filho da Natureza embrenhar-me-ia por essas matas seculares; contemplaria as maravilhas de Deus; veria o sol erguer-se no mar de ouro...Ouviria o murmúrio das ondas e o eco profundo e solene das florestas. E, se tudo isto não me inspirasse umapoesia nova, quebraria a minha pena com desespero, mas não a mancharia numa poesia menos digna do meubelo e nobre país.”(ALENCAR, José de. Crítica à Confederação dos Tamoios. Rio de Janeiro, Aguillar, 1965)Assinale o termo que não representa a mesma função sintática dos demais.a) “me”b) “o sol” Solução:c) “tudo isso”d) “poeta”

20) ESFAO - “A idéia da decadência do livro e da palavra impressa formou-se em grande parte emconseqüência do êxito da imagem e dos meios de comunicação que se servem da imagem: o cinema, atelevisão, a publicidade, os sistemas de sinalização, etc.”A partícula se é, respectivamente:1) partícula apassivadora - objeto indireto reflexivo;2) objeto direto reflexivo - sujeito;3) partícula apassivadora - objeto direto reflexivo;4) objeto indireto reflexivo - índice de indeterminação do sujeito;5) índice de indeterminação do sujeito - partícula apassivadora.

21) AFA - Este é o longo discurso que se vai pronunciar na Ordem dos Economistas.Na oração sublinhada analisam-se a voz, o sujeito, o agente e o paciente, respectivamente como:a) reflexiva, que, que, que;b) ativa, indeterminado, indeterminado, que;c) passiva sintética, que, indeterminado, que;d) passiva sintética, indeterminado, indeterminado, que.

22) AFA - “Que me conste, ainda ninguém relatou o seu próprio delírio; faça-o eu, e a ciência mo agradecerá.Se o leitor não é dado à contemplação destes fenômenos mentais, pode saltar o capítulo; vá direto ànarração.”(Machado de Assis. Memórias Póstumas de Brás Cubas)A declaração verdadeira a respeito dos elementos do texto está na alternativa:a) Em “é dado”, a voz passiva e o agente, indeterminado.b) Em “mo”, o “o” é pronome pessoal oblíquo com função de objeto direto.c) O vocábulo “direto” assumiria a forma feminina, se o referente fosse “leitora”.d) Em “é dado”, o verbo “ser” é de ligação, e “dado” é um adjetivo com função de predicativo.

23) AFA

Page 42: 1500 Port & Mat

Gigante pela própria natureza.És belo, és forte, impávido colosso,E o teu futuro espelha essa grandeza.Deitado eternamente em berço esplêndido,Ao som do mar e à luz do céu profundo,Fulguras, ó Brasil, florão da América,Iluminado ao sol do Novo Mundo!- Brasil de amor eterno seja símboloO lábaro que ostentas estrelado,E diga o verde-louro dessa flâmula- Paz no futuro e glória no passado!

Os sujeitos de espelha (3ª), fulguras (6ª) e seja (9ª) são, respectivamente:a) grandeza, tu, lábaro;b) futuro, Brasil, Brasil;c) Brasil, indeterminado, você;d) futuro, florão da América, Brasil.

24) Universidade Gama Filho - Qual a expressão grifada que não funciona como sujeito:a) “Estaria Arnaldo naquele instante dilacerado pelos estrepes (...)?”;b) “estrepes sobre que talvez o arremessara a queda desastrada”;c) “Flor invocara a intercessão daquele que para ela tudo podia na terra”;d) “Digo-lhe eu, Arnaldo Louredo que nunca menti a homem”;e) “o movimento de generosa simpatia e fraternidade que despertara em sua alma a tristeza do boi vencido”.

25) EFOMM - Estabeleça correspondência da coluna direita com a esquerda e, em seguida, assinale a opçãocorreta quanto à palavra se:1)Come-se bem naquele restaurante( )pronome apassivador2)Foi-se embora( )indeterminação do sujeito3)Ele não se deu descanso( )partícula expletiva4)Levantou-se e saiu( )objeto direto5)Cumpra-se o regulamento.( )objeto indireto

a) 5 - 1 - 4 - 3 - 2;b) 2 - 5 - 4 - 3 - 1;

Page 43: 1500 Port & Mat

c) 3 - 2 - 1 - 5 - 4;d) 5 - 1 - 2 - 4 - 3;e) 4 - 5 - 3 - 2 - 1.

26) Magistério - RJ - Faz-se erroneamente a classificação sintática do pronome relativo, na opção:a) “Mas eu não sei o que sou (...)” (predicativo)b) “O abismo que há entre o espírito e o coração.” (objeto direto)c) “O caderno que você desejava que eu trouxesse é caro.” (objeto direto)d) “O peão acabara de selar o cavalo, que puxara para fora da ramada.” (objeto direto)e) “Esclareceu que adiava a aquisição de uma chaleira de que andava muito necessitado” (objeto indireto)

27) Magistério - RJ - “criadas que se dão pressa em responder às visitas que a senhora saiu”A alternativa em que o se tem o mesmo valor sintático do exemplo acima é:a) “tema que se propôs elucidar”b) “Vêem-se ombros frágeis, quase de vidro”c) “desde que nelas não seja obrigado a se sentar”d) “O taberneiro se arrependeu de sua falta de sagacidade”e) “O céu estava azul, a paisagem estendia-se imensa e tranqüila”

28) Magistério - RJ - Há objeto direto e indireto na opção:a) “Há de no-la contar logo”b) “mas não me satisfazem os informes da História”c) “Nenhum de nós se importaria muito com a verdade”d) “A mocidade precisa transviar-se de alguma maneira”e) “Até os colegas perdiam a paciência com as suas dúvidas.

29) Catanduva - “O medíocre se deu ares de campeão.” O pronome se, nesta oração, é:a) pronome apassivador;b) expletivo;c) pronome indeterminador do sujeito;d) objeto direto;e) objeto indireto.

30) PUC - Assinale a opção na qual o pronome relativo que não exerce a função de objeto direto:a) “Aquele boi que ele tinha ao arção da sela”b) “era o prazer que D. Flor ia ter vendo o valente barbatão marcado com o seu ferro”c) “era finalmente a satisfação do velho capitão-mor, que se encheria de orgulho”d) “o movimento de generosa simpatia e fraternidade que despertara em sua alma a tristeza do boi vencido”e) “Apeou-se e tirou um ferro de marca, da maleta de couro, que trazia à garupa”

31) EFOMM - Assinale a opção em que a expressão grifada tem a função de complemento nominal:

Page 44: 1500 Port & Mat

a) A curiosidade do homem incentiva-o à pesquisa.b) O receio de errar dificulta o aprendizado das línguas.c) O respeito ao próximo é dever de todos.d) A cidade de Paris merece ser conhecida por todos.e) O coitado do velho mendigava pela cidade.

32) ESPCEX - Assinale a alternativa que contém erro de análise sintática.a) Em “A mãe adorava aos filhos”, a expressão “aos filhos” é objeto indireto.b) A frase “compram-se casas” está na voz passiva.c) Na oração “Tolos, já não os há”, “os” é objeto direto pleonástico.d) Em “Correr é bom para a saúde”, “bom” é predicativo do sujeito.Resposta: __________

33) AMAN - “O bom funcionário não soube resolvê-la, ninguém na repartição o soube.”Quanto aos pronomes deste período, para sermos exatos e completos, devemos afirmar que:a) todos estão empregados como objetos diretos;b) um é objeto direto; outro, indireto, ambos oblíquos;c) um é sujeito; os outros, objetos diretos;d) um é sujeito; o outro, objeto direto;e) os três são objetos diretos dos verbos saber e resolver.

34) BANESPA - Assinale a alternativa em que o termo grifado é complemento nominal:a) A enchente alagou a cidade.b) Precisamos de mais informações.c) A resposta ao aluno não foi convincente.d) O professor não quis responder ao aluno.e) Muitos caminhos foram abertos pelos bandeirantes.

35) FASP - “Uma lagartixa passou correndo à sua frente e sumiu-se por entre as macegas.” A palavra se é:a) pronome reflexivo e objeto direto;b) pronome reflexivo recíproco e objeto direto;c) partícula de realce - sem função sintática;d) pronome pessoal oblíquo e objeto direto.

GABARITO001) C002) A003) D004) C005) 3 - 1 - 2006) 102(2+4+32+64)

Page 45: 1500 Port & Mat

007) B008) E009) A010) D011) A012) B013) C014) A015) B016) A017) A018) D019) D020) B021) C022) D023) A024) C025) D026) E027) A028) A029) E030) C031) B032) A033) C034) C035) C

01) ESPCEX - No período: “... no fundo eu não estava triste com a viagem de meu pai, era a primeira vezque ele ia ficar longe de nós por algum tempo ...”, a oração sublinhada é:a) subordinada substantiva predicativa;b) subordinada adjetiva restritiva;c) subordinada adverbial de lugar;d) subordinada substantiva subjetiva.

02) ESFAO - Somando os números correspondentes às orações corretas quanto à classificação das mesmas,você encontrará a resposta da questão.“Garantiram-me que, depois de preenchido o formulário, que me enviaram pelo correio na segunda-feira semfalta e pagar a minha taxa de inscrição, eu seria atendido em menos de quarenta e oito horas.”(F. Sabino)(02) 1º oração: principal;(08) 2º oração: subordinada substantiva objetiva direta;(14) 3º oração: subordinada substantiva objetiva direta;

Page 46: 1500 Port & Mat

(20) 4º oração: subordinada adjetiva restritiva;(26) 5º oração: coordenada sindética aditiva em relação à 3º e subordinada adverbial temporal em relação à 1ª.

a) 24b) 36c) 48d) 56e) 70

03) AFA - Em que alternativa, a oração subordinada não é da mesma natureza da que existe em “Quero quevocês escrevam uma composição”?a) “E anunciou que não nos faria cantar.”b) “Esperava um irmão que vinha buscá-la.”c) “Vamos fazer de conta que estamos na aula de Português.”d) “Circulava a história de que ela dormia no sótão do colégio.”

04) EFOMM - Assinale o par de orações grifadas cuja classificação está trocada:a) Vi onde ela estuda. (subordinada substantiva objetiva direta)É sabido onde ela estuda. (subordinada substantiva subjetiva)b) Não chores, porque amanhã será um novo dia. (coordenada sindética explicativa)Não chores porque erraste o problema. (subordinada adverbial causal)c) Descobriu-se por quem o carro foi consertado. (subordinada adjetiva restritiva)Descobriu-se a pessoa por quem o carro foi consertado. (subordinada substantiva subjetiva)d) “Quando você foi embora,Fez-se noite em meu viver (...)” (subordinada adverbial temporal)Perguntei ao professor quando faríamos a prova. (subordinada substantiva objetiva direta)e) “Estêvão ficou ainda algum tempo encostado à cerca na esperança de que ela olhasse (...)” (subordinadasubstantiva completiva nominal)“A ambição e o egoísmo se opõem a que a paz reine sobre a Terra.” (subordinada substantiva objetivaindireta)

05) Colégio NavalVamos até a Matriz de Antônio Dias onde repousa, pó sem esperança, pó sem lembrança, o Aleijadinho.Vamos subindo em procissão a lenta ladeira.Padres e anjos, santos e bispos nos acompanhame tornam mais rica, tornam mais grave a romaria de assombração.Mas já não há fantasmas no dia claro,tudo é tão simples,tudo tão nu,as cores e cheiros do presente são tãofortes e tão urgentes que nem se percebem catingas e rouges, boduns e ouros do século 18.(O vôo sobre as igrejas, Carlos Drumond de Andrade)O “que” do verso 10 apresenta o valor semântico de:a) explicação;b) condição;c) conformidade;d) conseqüência;

Page 47: 1500 Port & Mat

e) lugar.

06) Colégio Naval - No trecho: “Todos diziam que ela era orgulhosa, mas afinal descobri que não”, a últimaoração se classifica como:a) coordenada sindética adversativa;b) principal;c) subordinada substantiva objetiva direta;d) subordinada adverbial comparativa;e) subordinada substantiva subjetiva.

07) AFASe o penhor dessa igualdadeConseguimos conquistar com braço forte,Em teu seio, ó Liberdade,Desafia o nosso peito a própria morte!- Mas, se ergues da justiça a clava forte,Verás que um filho teu não foge à luta,Nem teme, quem te adora, a própria morte,

As orações “Desafia o nosso peito a própria morte”, “que um filho teu não foge à luta” e “quem te adora”classificamse,respectivamente, como:a) principal, subordinada substantiva subjetiva, subordinada adjetiva restritiva;b) principal, subordinada adverbial temporal, subordinada substantiva objetiva direta;c) principal, subordinada substantiva objetiva direta, subordinada substantiva subjetiva;d) coordenada assindética, subordinada substantiva objetiva direta, subordinada substantiva apositiva.

08) EPCAR - Marque a alternativa que contém oração subordinada substantiva completiva nominal.a) “Como fazem os pelintras de hoje para não molhar os pés nos dias de chuva?”b) “Veio-me a desagradável impressão de que todo mundo reparava nas minhas galochas.”c) “Um dia as galochas me serão úteis, quando eu for suficientemente velho para merecê-las.”d) “No restaurante, onde entrei arrastando os cascos como um dromedário, resolvime ver livre dasgalochas.”e) “No centro da cidade um sol radioso varava as nuvens e caía sobre a rua, enchendo tudo de luz, fazendoevaporar as últimas poças de água que ainda pudessem justificar minhas galochas.”

09) EFOMM - Assinale o único exemplo em que não ocorre oração subordinada substantiva subjetiva:a) “Cansativo que seja, urge atravessarmos o campo que banha o Rio Negro antes de anoitecer.”b) “Todo escritor que surge reage contra os mais velhos, mesmo que o não perceba, e ainda que os admire.”c) “Dormiram naquilo, tinham-se acostumado, mas seria mais agradável dormirem numa cama de lastro decouro.”d) “É preciso que o pecador reconheça ao menos isto: que a Moral católica está certa e é irrepreensível.”e) “Sobre a multiplicidade informe e confusa dos bens da matéria é mister que paire a força ordenadora do

Page 48: 1500 Port & Mat

espírito.”

10) Colégio Naval - Somos uma pequena parte do elo, o miolo de envoltórios descomunais quedesconhecemos,arrogantes embora, na suposição de que é conosco que Deus se preocupa.A última oração do texto deve ser classificada como subordinada:a) adverbial concessiva;b) substantiva completiva nominal;c) adjetiva restritiva;d) substantiva predicativa;e) substantiva subjetiva.

11) ESFAO - Em “Dentro dela se abrigava a multidão de bárbaros e de estranhos ali recebidos com brandurae carinho” e “Tudo o que era natureza tinha o aspecto sinistro, trágico, desolador (...)”, temos,respectivamente:a) uma oração com sujeito simples; / duas orações com sujeito representado por pronomes (respectivamente,demonstrativo e relativo);b) duas orações, uma com sujeito claro, outra, oculto; / duas orações, tendo a primeira o sujeito simplesrepresentado por pronome relativo, a segunda, por um substantivo;c) uma oração com sujeito composto cujos núcleos são bárbaros e estranhos; / duas orações, estando asubordinada com sujeito oculto;d) uma oração com sujeito simples; / uma oração com sujeito representado por pronome indefinido;e) uma oração com sujeito pronominal; / uma oração com sujeito oracional.

12) EFOMM - “Não sei de onde te conheço.”A classificação correta da oração grifada está na opção:a) substantiva predicativa;b) adjetiva restritiva;c) substantiva subjetiva;d) substantiva objetiva indireta;e) substantiva objetiva direta.

13) EPCARQuando uma nuvem nômade destilagotas, roçando a crista azul da serra,umas brincam na relva, outras tranqüilas,serenamente entranham-se na terra.E a gente fala da gotinha que errade folha em folha e, trêmula, cintila,mas nem se lembra da que o solo encerra,de que ficou no coração da argila!Quanta gente, que zomba do desgostomudo, da angústia que não molha o rostoe que não tomba, em gotas, pelo chão

Page 49: 1500 Port & Mat

havia de chorar, se adivinhasseque há lágrimas que correm pela facee outras que rolam pelo coração!(Guilherme de Almeida)

Entre as alternativas abaixo, a única correta é:a) não há oração adverbial no texto em apreço;b) há menos de quatro orações adjetivas no soneto;c) há oração substantiva sem sujeito;d) na oração “que há lágrimas”, o que não é integrante;e) não há pronome demonstrativo no referido texto.

14) CESGRANRIO - “Hoje, a dependência operacional está reduzida, uma vez que o Brasil adquiriuautosuficiência na produção de bens como papel-imprensa (...)” A oração grifada no período acima tem valor:a) condicional;b) conclusivo;c) concessivo;d) conformativo;e) causal.

15) Colégio Naval“No entanto parece que os freqüentadores deste cinema Estão perfeitamente deslembrados de que terão demorrer - Porque em toda sala escura há um grande ritmo de esquecimento e equilíbrio.”A última oração do poema tem valor:a) subordinativo, revelando uma idéia de causa;b) coordenativo, traduzindo uma idéia de explicação;c) subordinativo, denotando conclusão;d) coordenativo, traduzindo uma idéia de tempo;e) subordinativo, revelando uma idéia de conseqüência.

16) UNIRIO - Assinale o item em que há uma oração adjetiva.a) Perdão, por Deus, perdão - respondeu o pombo.b) A pombinha, que era branca sem exagero, arrulhava, humilhada e ofendida com o atraso.c) Perdeste a noção do tempo?d) A tarde era tão bonita que eu tinha de vir andando.e) O pombo caminhava pelo beiral mais alto, do outro lado. Um pouco além, gritavam as gaivotas.

17) Colégio NavalNada sei, afinal, da tua aparência no tempo, a não ser o que me contavam em casa, desde menino: que erasruivo como eu, que vieste em vinte e quatro, com os primeiros colonos, e abandonaste logo a tua pobrelavoura, encravada nos matos de Sapucaia, para alistar-te entre os Farroupilhas.Pudesse eu, armado de vidência, acompanhar-te o passo, Maria Klinger; ver claramente vistas as tuasandanças de colona; como venceste as veredas e picadas; como tomaste o caminho que ia dar nos arredores

Page 50: 1500 Port & Mat

da cidade; como paraste, cansada, à sombra das árvores, ou foste pedir, na tua língua de trapos, um pouco deágua para a tua sede (...)Assinale o único item que não apresenta uma oração subordinada substantiva objetiva direta.a) “(...) a não ser o que me contavam em casa (...)”b) “(...) que eras ruivo como eu.”c) “(...) e abandonaste logo a tua pobre lavoura (...)”d) “(...) como venceste as veredas e picadas (...)”e) “(...) ou foste pedir (...) um pouco de água para a tua sede”

18) PUC - “É preciso (I) levar tudo isso em conta (II) quando se analisa o (III) que está ocorrendo em nossosdias.”A classificação das orações subordinadas sublinhadas é, respectivamente:IIIIIIa)adjetivaadverbialsubstantiva;b)substantivaadjetivasubstantiva;c)adverbialsubstantivaadjetiva;d)substantivaadverbialadjetiva;e)adverbialadverbialsubstantiva.

19) ESPCEX - Marque a alternativa que indica a correta classificação das orações sublinhadas, segundo aordem em que estas aparecem nas frases abaixo:1) Robertinho, com ser inteligente, não foi aprovado no concurso.2) Não é permitido transitar por esta rua.3) Chocou-nos o seu modo áspero de falar, embora não tivesse o propósito de ofender a pessoa alguma.a) subordinada substantiva apositiva, subordinada substantiva completiva nominal, subordinada adjetiva;b) subordinada adverbial conformativa, subordinada substantiva predicativa, subordinada completivanominal;c) subordinada adverbial concessiva, subordinada substantiva subjetiva, subordinada substantiva

Page 51: 1500 Port & Mat

completiva nominal;d) subordinada substantiva apositiva, subordinada substantiva subjetiva, subordinada adjetiva.Resposta: __________

20) Colégio Naval - No período: “Quando o rei Herodes mandou decapitar crianças, eu o levei na fuga parao Egito”, as orações classificam-se, respectivamente:a) subordinada adverbial temporal / subordinada substantiva objetiva direta / principal;b) subordinada adverbial temporal / principal;c) principal / substantiva objetiva direta / coordenada assindética;d) coordenada sindética conclusiva / coordenada assindética;e) subordinada adverbial proporcional / principal.

21) UNIRIO - Em “Entende-se bem que D. Tonica observasse a contemplação dos dois”. à oração principalsegue-se uma oração subordinada:a) substantiva subjetiva;b) substantiva objetiva direta;c) adjetiva restritiva;d) adverbial causal;e) adverbial concessiva.

22) ESFAO - Que oração subordinada substantiva em destaque é completiva nominal:1) desejo que um dia me restitua uma parte de sua estima.2) habituei-me a considerar a riqueza primeira força.3) pensando que os poderia refazer mais tarde.4) e os exemplos ensinavam-me que o casamento era meio legítimo.5) o casamento era meio legítimo de adquiri-la.

23) EFOMM - Marque a classificação correta das orações destacadas no período: “Ao analisar odesempenho da economia brasileira, os empresários afirmaram que a produção e o lucro eram bastanterazoáveis.”a) subordinada adverbial temporal - subordinada substantiva objetiva direta;b) principal - subordinada substantiva completiva nominal;c) subordinada adverbial temporal - subordinada adjetiva restritiva;d) principal - subordinada adverbial final;e) subordinada adverbial condicional - subordinada substantiva subjetiva.

24) Colégio Naval - Marque a alternativa em que a oração destacada não se encontra corretamenteclassificada.a) “Parece que eu não acreditava na história” - oração subordinada substantiva subjetiva;b) “(...) torcíamos para ele subir mais” - oração subordinada adverbial final;c) “Lembro-me (...) desse jardim que não existe mais.” - oração subordinada adjetiva restritiva;d) “Lá fora, uma galinha cacareja, como antigamente.” - oração subordinada adverbial comparativa;

Page 52: 1500 Port & Mat

e) “Diziam que São Pedro estava arrastando os móveis” - oração subordinada substantiva subjetiva.

25) UNIRIO - No período “Ah, arrulhou de repente a pomba, quando distinguiu, indignada, o pombo quechegava (...)”, as duas orações subordinadas são respectivamente:a) adjetiva e adverbial temporal;b) substantiva predicativa e adjetiva;c) adverbial temporal e adverbial temporal;d) adverbial temporal e adverbial consecutiva;e) adverbial temporal e adjetiva.

26) EFOMM - Assinale a opção em que uma oração subordinada destoa das demais:a) Nunca souberam como ele morreu.b) É proibido falar ao motorista.c) Diz-se que amor com amor se paga.d) Nunca se sabe quando ele fala sério.e) Importa apenas que os dois se respeitem.

27) UFRRJ - “Tal era a fúria dos ventos, que as copas das árvores beijavam o chão.” Neste período, a oraçãosubordinada é adverbial:a) concessiva;b) condicional;c) consecutiva;d) proporcional;e) final.

28) EFOMM - “Depois que o velho morresse, não teria mais graça saltar o muro para roubar fruta na suahorta.”As duas últimas orações do período são, respectivamente:a) subordinada substantiva subjetiva / subordinada substantiva completiva nominal;b) subordinada substantiva objetiva direta / subordinada adverbial final;c) subordinada substantiva objetiva indireta / subordinada substantiva completiva nominal;d) subordinada substantiva subjetiva / subordinada adverbial final;e) subordinada substantiva predicativa / subordinada completiva nominal.

29) CESGRANRIO - Assinale a classificação correta da oração sublinhada:“Caíra no fim do pátio, debaixo de um juazeiro, depois tomara conta da casa deserta.”a) subordinada adverbial temporal;b) subordinada adverbial proporcional;c) subordinada adverbial consecutiva;d) coordenada sindética conclusiva;e) coordenada assindética.

Page 53: 1500 Port & Mat

30) Colégio Naval - No período: “E era uma tal multidão de astros a tremeluzir que, juro, às vezes, tinha aimpressão de ouvir o burburinho infantil de suas vozes.”, o vocábulo sublinhado introduz uma oração:a) subordinada adjetiva explicativa;b) subordinada adverbial causal;c) subordinada substantiva objetiva direta;d) subordinada adverbial consecutiva;e) subordinada adverbial concessiva.

31) PUC - “quando eu quiser sei onde achá-lo”As orações sublinhadas são classificadas, respectivamente, como:a)adverbialadjetiva;b)adverbialadverbial;c)adverbialsubstantiva;d)adjetivasubstantiva;e)principaladverbial.

32) EFOMM - Todas as orações estão analisadas corretamente, exceto:a) Sem que me ajudasses, nada poderia fazer. (sub. adverbial condicional)b) Os empregados estavam esgotados de modo que se retiraram imediatamente. (sub. adverbial consecutiva)c) Admira-me que não tenhas podido chegar a tempo. (sub. substantiva subjetiva)d) “Plante, que o João garante.” (coordenada sindética explicativa)e) Fazia um calor de fritar ovos no chão. (sub. substantiva completiva nominal)

33) ESFAO - Marque a opção correta:Comparando-se as duas falas de Esopo:1º “Com a língua se ensina, se persuade ... se afirma.”2º “É a língua que mente, que esconde ... que corrompe.”

Verifica-se na estruturação a seguinte característica:a) apenas períodos compostos por subordinação;b) na primeira, um período composto por coordenação; na segunda, um período composto por subordinação;c) orações sem sujeitos, pois todos os verbos são impessoais;d) identidade sintática, mas oposição semântica;e) semelhança semântica, sintática e morfológica.

34) Colégio Naval“Sai, afastando-me dos grupos, e fingido ler os epitáfios. E, aliás, gosto dos epitáfios; eles são, entre a gente

Page 54: 1500 Port & Mat

civilizada, uma expressão daquele pio e secreto egoísmo que induz o homem a arrancar à morte um farrapoao menos da sombra que passou. Daí vem, talvez, a tristeza inconsolável dos que sabem os seus mortos navala comum; parecelhes que a podridão anônima os alcança a eles mesmos.”(Quincas Borba - M. de Assis)“(...) que a podridão anônima os alcança a eles mesmos.”uma oração:a) adjetiva restritiva;b) adjetiva explicativa;c) adverbial condicional;d) substantiva subjetiva;e) substantiva objetiva direta.

35) UNIRIO - “(...) fi-la construir de propósito, levado de um desejo tão particular que me vexa imprimi-lo,mas vá lá.”O vocábulo sublinhado introduz oração que denota:a) tempo;b) causa;c) condição;d) comparação;e) conseqüência.

GABARITO001) D002) D003) B004) C005) D006) C007) C008) B009) B010) B011) A012) E013) C014) E015) A016) B017) A018) D019) C020) A021) A022) 5023) A024) E

Page 55: 1500 Port & Mat

025) E026) A027) C028) D029) E030) D031) C032) E033) B034) D035) E

(Analista Com.Exterior/1998) Assinale o período com grafia inteiramente correta.a) Na solenidade de inauguração da escola, o diretor revelou alviçareira notícia: a liberação de recursos paraa construção de uma quadra poliesportiva.b) Agradeceu também aos professores e estudantes que sempre perfilharam ao lado da direção na busca deum ensino de melhor qualidade.c) Renovou seu idealismo de puguinar para a escola alçarse à categoria de escola-padrão do município.d) Enfatizou serem as escolas, ocupem-se elas do grau de ensino que for, o lugar em que se vai construindo ocidadão consciente e crítico.e) Reinterou sua convicção no atingimento das metas a que se propôs como fundador e primeiro diretor daescola.

GABARITO: D

(MPU/2005) Marque o item em que uma das sentenças não está gramaticalmente correta.a) A literatura depende muito de condições subjetivas, raramente satisfaz apenas os sentidos, exigecolaboração, embora muitos acreditem que as obras literárias possam brotar de cérebros insulados. / Aliteratura depende muito de condições subjetivas, raramente satisfaz apenas aos sentidos, exige colaboração,embora muitos acreditem que as obras literárias possam brotar de cérebros insulados.

b) Um povo não perde os seus mais fortes determinantes se recebe, aceita e pratica a pintura e a música deoutra origem, mas dificilmente adotará literatura estranha sem perda de alguns de seus valores. / Um povonão perderá os seus mais fortes determinantes se receber, aceitar e praticar a pintura e a música de outraorigem, mas dificilmente adotará literatura estranha sem perda de alguns de seus valores.

c) Já tive ocasião de mostrar quanto me parecem precárias três afirmativas de Euclides da Cunha: a questãodo cruzamento; a fatalidade da luta das raças e o autoctonismo do homem americano. / Já tive ocasião demostrar como me parecem precárias três afirmativas de Euclides da Cunha: a questão do cruzamento; afatalidade da luta das raças e o autoctonismo do homem americano.

d) Quando surgiu Euclides da Cunha, nossa literatura podia enumerar grandes nomes pertencentes ao“sistema” de que falei há pouco. / Quando surgiu Euclides da Cunha, nossa literatura podia enumerargrandes nomes pertencentes ao “sistema” de que faz pouco falei.

e) No Brasil, a nacionalidade e a literatura formaram um “sistema” interessantíssimo, que a cerca de

Page 56: 1500 Port & Mat

trezentos anos desenvolve-se. / No Brasil, a nacionalidade e a literatura formaram um “sistema”interessantíssimo, que há cerca de trezentos anos se desenvolve.

(Baseado em Roquette Pinto)

GABARITO: E

(TRF/2002) Indique a opção que completa com correção gramatical e com coerência as lacunas do textoabaixo.O Estado cresceu em termos de pessoal e, principalmente, em termos de receita e despesa. Em muitospaíses, os servidores públicos, excluídos os trabalhadores das empresas estatais, correspondem___________10 a 20 por cento da força de trabalho, __________no início do século XX essa proporção estavapróxima de 5 por cento. As despesas do Estado, por sua vez, ___________ nesse período: nos últimos trintaanos ____________, variando entre 30 e 50 por cento do PIB. Naturalmente, esse processo de crescimentoocorria ao mesmo tempo em que _________________as funções do Estado, principalmente na área social.

(Luiz Carlos Bresser Pereira, com adaptações)

a) a cerca de / enquanto / multiplicaram-se / dobraram / se ampliavamb) acerca de / quando / multiplicara / dobraram-se /ampliava-sec) cerca de / quanto / multiplicaram / dobravam-se / se ampliavad) em cerca de / em quanto / se multiplicava / dobrou / ampliavame) de cerca de / por quanto / multiplicavam / dobravam-se / ampliava

GABARITO: A

(Auditor de Fortaleza/1998) Nas questões seguintes, indique entre os itens sublinhados o que contém errogramatical ou impropriedade vocabular.Não é muito difícil imaginar os motivos que teria a Argentina para entabular(A) agora uma conversa maisséria com o Brasil a cerca(B) da criação de uma moeda única no Mercosul.Existe uma motivação não reconhecida oficialmente mas plenamente visível: o receio das desvalorizações doreal – na base de(C) 0,6% ao mês – e o impacto que isso já está tendo sobre os preços das exportaçõesargentinas ao mercado brasileiro. O tema é complexo e tem muito mais implicações(D) do que a decisão demandar um argentino em viagemà(E) Lua.(Maria Clara R. M. Prado - Gazeta Mercantil - 21 e 22/2/1998,adaptado)a) Ab) Bc) Cd) De) E

GABARITO: B

Page 57: 1500 Port & Mat

(AFC/SFC/2000) Assinale, entre as substituições propostas, a que corrige adequadamente o erro do trechoseguinte:Antes de digladiar sobre como os novos impostos deveriam ser distribuídos entre as três esferas de governo -questão que consumiu a melhor parte dos humores e energia de prefeitos,governadores e autoridadesfazendárias federais nos últimos três anos -, talvez vale a pena perguntar como, de um ponto de vistaagregado, deixando por um momento de lado as complexidades do federalismo fiscal, poderiam ser os R$ 131bilhões arrecadados de forma mais racional.(Adaptado de Rogério L. F. Werneck , O Estado de S. Paulo, 27/10/2000)

SUBSTITUIR PORa) Digladiar degladiarb) Sobre a cerca da formac) Entre dentred) Vale valhae) poderiam ser poderia ser

GABARITO: D

(TCE RN/2000) Marque o item em que um dos dois períodos está gramaticalmente incorreto.a) Entre as leis editadas pela União, algumas há que se destinam à organização político-administrativa doEstado brasileiro, penetrando na estrutura da República Federativa,para nela dispor instituições einstitutos, quer essenciais,quer acidentais à república e à federação. / Entre as leis editadas pela União,algumas há que se destinam à organização política-administrativa do Estado brasileiro, penetrando naestrutura da República Federativa, para nela dispôr instituições e institutos, quer essenciais, quer acidentaisà república e à federação.

b) O federalismo brasileiro é de duplo grau, declinando por dois degraus entre três patamares, pelo quesuporta, em três níveis de poder, três repartições genéricas de compe- tência./ O federalismo brasileiro é deduplo grau, declinando por dois degraus entre três patamares, razão por que suporta,em três níveis de poder, três repartições genéricas de competência.

c) No gênero das leis federativas, é possível discernir duas espécies bem visíveis: leis federais intransitivas etransitivas. / No gênero das leis federativas, podem-se discernir duas espécies bem visíveis: leis federaisintransitivas e transitivas.

d) As leis nacionais podem ser de ordem pública ou de ordem privada, guardando preponderante interessepúblico ou administrativo, ou social, ou privado. / As leis nacionais podem ser de ordem pública ou de ordemprivada, e guardam preponderante interesse público ou administrativo, ou social, ou privado.e) Algumas leis eminentemente federativas, como o Código Tributário Nacional, autoproclamam-se‘nacionais’. /Algumas leis eminentemente federativas, como o Código Tributário Nacional, seautoproclamam ‘nacionais’.(Baseado em Sérgio Resende de Barros)

Page 58: 1500 Port & Mat

GABARITO: A

(TTN 1998) Leia o texto seguinte e responda à questão abaixo.Segundo os cientistas, os macacos pongídeos – termo usado para designar os bichos mais próximos dohomem, como gorilas, orangotangos, chimpansés e bonobos – vivem em sociedades organizadas, em que asrelações entre os individuos são semelhantes às humanas. Diferentemente dos animais mais primitivos, aosquais se atribuem apenas emoções simples, como medo e fome, esses macacos parecem possuir compaixão esolidariedade. Sua capacidade intelectual está muito acima de todo o restante dos bichos e a diferença entreseu código genético e o dos sereshumanos é de apenas 1,6%. As pesquisas com essas espécies vem causando tanta comoção que, no final de1997, um grupo de biólogos, assessorado por advogados ambientalistas,publicou um documento pedindo aextensão dos direitos humanos aos pongídeos!(Valéria França - Veja - 28/1/98, adaptado)

Há, no texto,a) três erros de ortografiab) nenhum erro de ortografiac) dois erros de ortografiad) um erro de ortografiae) quatro erros de ortografia

GABARITO: A

(AFRF 2003) Indique o item em que todas as palavras estão corretamente empregadas e grafadas.a) A pirâmide carcerária assegura um contexto em que o poder de infringir punições legais a cidadãosaparece livre de qualquer excesso e violência.b) Nos presídios, os chefes e subchefes não devem ser exatamente nem juízes, nem professores, nemcontramestres,nem suboficiais, nem “pais”, porém avocam a si um pouco de tudo isso, num modo deintervenção específico.c) O carcerário, ao homogeinizar o poder legal de punir e o poder técnico de disciplinar, ilide o que possahaver de violento em um e de arbitrário no outro, atenuando os efeitos de revolta que ambos possam suscitar.d) No singular poder de punir, nada mais lembra o antigo poder do soberano iminente que vingava suaautoridade sobre o corpo dos supliciados.e) A existência de uma proibição legal cria em torno dela um campo de práticas ilegais, sob o qual se chega aexercer controle e aferir lucro ilícito, mas que se torna manejável por sua organização em delinqüência.(Itens adaptados de Michel Foucault)

GABARITO: B

(MPOG/ 2003) Assinale a opção que corresponde a erro gramatical ou de grafia das palavras.Considerando que a constituição de uma nova cultura do trabalho nos empreendimentos populares só pode

Page 59: 1500 Port & Mat

ser(1) compreendida como um processo que perspassa(2) o conjunto mais amplo das relações sociais, seria(3)uma ilusão imaginar que é possível encontrar no interior da sociedade capitalista uma organizaçãoeconômica que, mesmo gerida(4) pelos próprios trabalhadores, pudesse se(5) caracterizar, em seu conjunto,como “cultura de novotipo”.(Adaptado de Lia Tiriba)a) 1b) 2c) 3d) 4e) 5

GABARITO: B

(AFC/CGU 2003/2004) Assinale a opção que corresponde a palavra ou expressão do texto que contraria aprescrição gramatical.No século XX, a arte cinematográfica introduziu um novo conceito de tempo. Não mais o conceito linear,histórico,que perspassa(1) a Bíblia e, também, as pinturas de Fra Angelico ou o Dom Quixote, de Miguel deCervantes. No filme,predomina a simultaneidade(2). Suprimem-se(3) as barreiras entre tempo e espaço. Otempo adquire caráter espacial, e o espaço, caráter temporal. No filme, o olhar da câmara e do espectador(4)passa, com toda a liberdade, do presente para o passado e, desse, para o futuro. Não há continuidadeininterrupta(5).(Adaptado de Frei Betto)a) 1b) 2c) 3d) 4e) 5]GABARITO: A

(AFTE RN/2005) Marque a assertiva errada em relação ao texto seguinte:O ano de 2003 é marcado pela recessão econômica no Brasil e em vários países do mundo. Aqui, o clima foide expectativa em relação a um novo governo que assumiu o País diante de um grave quadro de desigualdadesocial. O Brasil assistiu, estarrecido, no outro lado do mundo, a uma invasão no Oriente Médio promovidapela dupla Bush/Blair. E o terrorismo só recrudesceu. De que forma todos esses acontecimentos podem terinfluído noimaginário do executivo brasileiro?(Carta Capital, n° 307)c) “Expectador” é uma palavra cognata de “Expectativa”, mas ao contrário dessa pode também ser grafadacom “s” na primeira sílaba, sem alteração de sentido.

Item INCORRETO.

Page 60: 1500 Port & Mat

(AFC/CGU 2003/2004) Assinale a opção que corresponde a palavra ou expressão do texto que contraria aprescrição gramatical.Aos poucos, o horizonte histórico apaga-se(1), como as luzes de um palco após o espetáculo. A utopia sai decena, o que(2) permite a Fukuyama vatiscinar(3): "A história acabou". Ao contrário do que(4) adverte Coélet,no Eclesiastes, não há mais tempo para construir e tempo para destruir; tempo para amar e tempo para odiar;tempo para fazer a guerra e tempo para estabelecer a paz. O tempo é agora. E nele se sobrepõem(5)construção e destruição, amor e ódio, guerra e paz.(Adaptado de Frei Betto)a) 1b) 2c) 3d) 4e) 5

GABARITO: C

(TTN/ 1997) Assinale o item incorreto em relação às exigências do padrão culto da língua escrita.a) A Organização Internacional de Aviação Civil (OECI) considera todo passageiro aéreo um terroristainocente em potencial.b) A entidade das Nações Unidas encarregada de cuidar da segurança de vôo, com sede no Canadá, emitiu,ano passado, alerta aos organismos governamentais sobre produtos perigosos carregados em bagagens. c) O alerta lista 3.291 itens e solicita que os governos criem campanhas publicitárias para concientizar ospassageiros sobre o risco de levar na bagagem explosivos que se escondem sob a forma de objetosaparentemente inofensivos, como sprays para cabelos, isqueiros a gás, caixas de fósforos, kits de limpeza decomputadores e pilhas alcalinas.d) A recomendação não pára aí. Alguns produtos podem causar irritação ou sufocamento, como ácidofórmico, arsênico e inseticidas, ou ferimentos graves, como oxidantes, ácido sulfúrico, produtos alcalinos esoda cáustica.e) Outro ponto lembrado na norma refere-se a interferências em equipamentos aeronáuticos causadas porímãs e equipamentos eletrônicos, como celulares.(Correio Braziliense - 15.7.97, com adaptações)

GABARITO: C

(TTN/1997) Assinale o item incorreto em relação às exigências do padrão culto da língua escrita.a) O Fundo de Estabilização Fiscal (FEF) permite ao governo remanejar recursos do orçamento e fazer umareserva para cobrir despesas extras. O FEF é composto de recursos diversos, entre eles parte do Imposto deRenda (IR) de funcionários públicos e fornecedores da União.b) Sem o FEF, o governo só pode realizar despesas previstas no orçamento, conforme as regras daConstituição.Sem o fundo não é possível usar para pagar salários um dinheiro destinado à saúde, por exemplo. O FEF,

Page 61: 1500 Port & Mat

portanto,garante maior mobilidade.c) Entretanto, o IR é também o principal item do Fundo de Participação dos Municípios (FPM), umaespécie de mesada paga pelo governo aos municípios.d) Por determinação da Constituição, todos os municípios têm direito a 17% do FPM (que reune recursos doIR e do Imposto sobre Produtos Industrializados).e) Desde o início da vigência do FEF, em 1994, (antes chamava-se Fundo Social de Emergência) osmunicípios já perderam R$ 1 bilhão/ano. Isso porque a principal fonte do FEF e do FPM é a mesma: o IR.(Correio Braziliense - 15.7.97, com adaptações)

GABARITO: D

(AFC/STN 2002) - Marque o item em que a forma proposta não preenche a lacuna do respectivo segmentodo texto com precisão vocabular e correção gramatical.a) No âmbito das políticas públicas, houve mudanças concretizadas através das propostas coletadas noRelatório do Ministério da Justiça pelo Comitê Nacional para a participação brasileira na III ConferênciaMundial das Nações Unidas contra o Racismo, --------------------- Racial, Xenofobia e IntolerânciaCorrelata.Descriminaçãob) No documento oficial está a seguinte proposta: “adição de cotas ou outras medidas afirmativas quepromovam --------- -------------------------------------- universidades públicas.” o ascenso de negros às c) O própriopresidente do Supremo Tribunal Federal defende a ação afirmativa, que considera constitucional:“Precisamos deixar de lado a postura ---------------------- e partir para atos concretos.” contemplativad) “O único modo de se ---------------------- desigualdades é colocar a lei a favor daquele que é tratado de mododesigual.” corrigireme) Em vários setores do Governo Federal, medidas de ação afirmativa já foram ou --------------------implantadas. têm sido (Baseado em UnB Revista, n° 6)

GABARITO: A

(TCE RN/2000) Marque o item em que um dos dois períodos está gramaticalmente incorreto.a) Entre as leis editadas pela União, algumas há que se destinam à organização político-administrativa doEstado brasileiro, penetrando na estrutura da República Federativa, para nela dispor instituições einstitutos, quer essenciais, quer acidentais à república e à federação. / Entre as leis editadas pela União,algumas há que se destinam à orga- nização política-administrativa do Estado brasileiro, penetrando naestrutura da República Federativa, para nela dispôr instituições e institutos, quer essenciais, quer acidentaisà república e à federação.b) O federalismo brasileiro é de duplo grau, declinando por dois degraus entre três patamares, pelo quesuporta, em três níveis de poder, três repartições genéricas de compe- tência./ O federalismo brasileiro é de duplo grau, declinando por dois degraus entre três patamares, razão por quesuporta,em três níveis de poder, três repartições genéricas de competência.c) No gênero das leis federativas, é possível discernir duas espécies bem visíveis: leis federais intransitivas etransitivas. / No gênero das leis federativas, podem-se discernir duas espécies bem visíveis: leis federaisintransitivas e transitivas.d) As leis nacionais podem ser de ordem pública ou de ordem privada, guardando preponderante interesse

Page 62: 1500 Port & Mat

público ou administrativo, ou social, ou privado. / As leis nacionais podem ser de ordem pública ou de ordemprivada, e guardam preponderante interesse público ou administrativo, ou social, ou privado.e) Algumas leis eminentemente federativas, como o Código Tributário Nacional, autoproclamam-se‘nacionais’. / Algumas leis eminentemente federativas, como o Código Tributário Nacional, seautoproclamam ‘nacionais’.(Baseado em Sérgio Resende de Barros)

AFRF 2003) Indique o item em que todas as palavras estão corretamente empregadas e grafadas.a) A pirâmide carcerária assegura um contexto em que o poder de infringir punições legais a cidadãosaparece livre de qualquer excesso e violência.b) Nos presídios, os chefes e subchefes não devem ser exatamente nem juízes, nem professores, nemcontramestres, nem suboficiais, nem “pais”, porém avocam a si um pouco de tudo isso, num modo deintervenção específico.c) O carcerário, ao homogeinizar o poder legal de punir e o poder técnico de disciplinar, ilide o que possahaver de violento em um e de arbitrário no outro, atenuando os efeitos de revolta que ambos possam suscitar.d) No singular poder de punir, nada mais lembra o antigo poder do soberano iminente que vingava suaautoridade sobre o corpo dos supliciados.e) A existência de uma proibição legal cria em torno dela um campo de práticas ilegais, sob o qual se chega aexercer controle e aferir lucro ilícito, mas que se torna manejável por sua organização em delinqüência.

(Itens adaptados de Michel Foucault)

(Procurador BACEN/2002) Julgue as afirmações a respeito do emprego das estruturas lingüísticas do texto.Do ponto de vista do “pai de família pobre” da década de 20 ou 30, o Estado aparece como aquele que deveprover os cidadãos do conforto material mínimo à sobrevivência, na forma de emprego ou de outras condiçõesmais diretas, como moradia, saúde ou educação. Não se trata de emitir um juízo de valor sobre estaconcepção, mas de constatar sua existência. Necessário, porém, confrontar isso com o reverso da medalha,ou seja, a política estatal em relação a esse tipo de reivindicação, especialmente para o período que antecede1930 e que surge para a historiografia comodomínio exclusivo das oligarquias.(Jaime Rodrigues, Crise da primeira república: classes médias e Estado na década de 20, com adaptações)

IV.A substituição de “esse tipo de reivindicação” (l.7-8) por essa reivindicação preservaria a coerênciatextual, mas, para manter a correção gramatical da oração, seria necessário empregar o sinal indicativo decrase no “a” que antecede a expressão.

Item INCORRETO.

(AFPS/2002) Identifique o item sublinhado que contenha erro de natureza ortográfica ou gramatical, ouimpropriedade vocabular.Fala-se(A) com arroubo(B) sobre os inesgotáveis recursos de novas tecnologias, como o vídeo ou a realidadevirtual, mas qualquer reflexão à respeito do(C) invariavelmente orbita(D) em torno da matériaprima(E)

Page 63: 1500 Port & Mat

desta página – o texto.(Paul Saffo, com adaptações)a) Ab) Bc) Cd) De) E

Gabarito: C

(Fiscal de Fortaleza/1998) Indique entre os itens sublinhados o que contém erro gramatical ou impropriedadevocabular.Geograficamente, a região entre o Parnaíba, o Tocantins e o São Francisco pertencem(A), em grande parte,a (B) Pernambuco, mas a história prende-a à(C) Bahia. Foram baianos que, procurando terrenos apropriadosà criação de gado, passaram à(D) Serra do Espinhaço, e favorecidos pelas catingas decíduas, chegaram ao riode São Francisco, espontando(E) todos os rios secos que retalham Pernambuco, Paraíba, Rio Grande doNorte e Ceará.(Capistrano de Abreu, adaptado)a) Ab) Bc) Cd) De) E

Gabarito: A (concordância)

(TTN/1998) Identifique o item sublinhado que contém erro de natureza ortográfica ou gramatical ouimpropriedade vocabular, e marque a letra correspondente.Só uma visita à(A) Cuba, a ilha comunista encravada no calcanhar dos Estados Unidos, poderia ter levadouma viagem de João Paulo II de volta às(B) manchetes com grande destaque. Numa ressurreição dointeresse despertado pelas primeiras viagens pontifícias, quando desafiou o império vermelho na Polônia erezou missa em grotões do Terceiro Mundo, o desembarque do Papa em Havana esteve envolto na místicade ser um desses momentos históricos, carregados de promessas. A Igreja sofre restrições em Cuba, que jáfoi um país católico e hoje conta com um número insignificante de seguidores da palavra de Roma, mas oPapa não foi à(C) ilha para passar um sermão público em Fidel Castro. Aos 71 anos e saúde debilitada, ocomandante Fidel não tem sucessor à(D) altura de seu carisma e o mundo do pós- comunismo tornaimpensável a manutenção do regime cubano, tal como sobrevive hoje, depois que Fidel for prestar contas a(E)Marx.

(Veja - 28/1/98, adaptado)a) A

Page 64: 1500 Port & Mat

b) Bc) Cd) De) E

Gabarito: A

(TRF/2000) Assinale a opção em que há erro gramatical. A economia brasileira, na década de 30 até ametade da de 40,embora(A) mais pujante(B) que a de alguns países vizinhos,poderia igualmente classificar-se como débil. Aquela altura(C) aindústria nacional produzia quase que só bens de consumo. Apesar de o(D) Brasil dispor(E) das maiores reservas mundiais do melhor minério de ferro, só em virtude da SegundaGuerra Mundial, sob gigantesco esforço, foi possível construir a Companhia Siderúrgica Nacional.(Sylvio Wanick Ribeiro, com adaptações)a) Ab) Bc) Cd) De) E

Gabarito: C

(TTN/1998) Marque o texto que contém erro de estruturação sintática ou de pontuação.a) Os profissionais liberais têm-se mostrado conscientes e dispostos a participar do movimento pela reformada sociedade.b) Para diminuir a sonegação fiscal, o governo concede anistia a quem apresentar a retificação de suadeclaração de renda.c) Cidadãos e governo colocaram-se frente a frente e finalmente entraram em acordo sobre a reformatributária.d) Devido a necessidade de tornar a tarefa política mais ética e saudável, tem havido significativamobilização.e) O Secretário solicita a essas pessoas que recorram a profissionais credenciados para obteresclarecimentos.

Gabarito: D

AFC SFC/2000) Indique a seqüência que preenche corretamente as lacunas do texto abaixo.A história nos mostra que o desenvolvimento econômico europeu, ____ partir das navegações, sempre se fez____custa dos territórios ultramarinos. Não foram apenas as matérias-primas, destinadas ao consumo ou____ produção que o financiaram, mas também o capital propriamente dito, fruto dos lucros e resultado dosaqueio da natureza virgem. Hoje, a biotecnologia abre grande perspectiva ___ um país como o Brasil, dericos bancos genéticos. O nosso território foi dos primeiros ____ ser saqueado em sua riqueza vegetal. Énecessário impedir que os produtos da flora e da fauna nos sejam roubados, como roubados fomos nopassado. No entanto, o governo está empenhado em aprovar uma proposta de emenda ____ Constituição

Page 65: 1500 Port & Mat

que facilitará a entrega de nossos recursos biológicos ____ estrangeiros.(Mauro Santayana, Correio Braziliense, 25/10/2000, com adaptações)a) a, à, à, a, a, à, ab) a, a, à, a, a, a, ac) a, à, à, à, a, à, àd) à, a, a, à, à, a, ae) à, à, a, à, a, à, a

Gabarito: A

(Procurador BACEN/2002)O ingresso dos bancos na era digital não se fez, obviamente, sem grandes e continuados investimentos.Sólida infraestrutura, bom trabalho de orientação, as experiências bem-sucedidas de quem não vê maioresdificuldades na operação eletrônica vão dissipando _____resistências dos ainda não-digitalizados. Osclientes adaptam-se ___ novas tecnologias de modos muito distintos. _____ segmentos de pessoas maduras,com mais de 60 anos, nos quais a utilização da Internet é maior do que em segmentos jovens, com menos de30 anos.Procura-se fornecer o maior número de informações aos clientes, ajudando-os ___ superar as primeirasdificuldades e demonstrando que, nos meios eletrônicos, "o índice de falhas sistêmicas é mínimo". Emborametade da população economicamente ativa brasileira esteja fora do sistema bancário – e este é um novoterritório ainda ___ conquistar –,___ marcha da digitalização para os que já estão dentro do sistema é umcaminho que não tem volta.(Adaptado de www2.estado.estadao.com.br/edição/especial/bancos)Indique a opção cujos itens completam corretamente as lacunas do texto acima.a) as às Há a a ab) as as Há a a àc) as às A à à ad) às às Há a à ae) às as A à a a

Gabarito: A

(AFRF 2002.1) Marque a opção que preenche corretamente as lacunas.completamente excluídos das engrenagens de desenvolvimento da sociedade, os miseráveis são reduzidos_____uma condição subumana.Seu único horizonte passa ____ ser ____ luta feroz pela sobrevivência. No lixão do Valparaíso, ____poucosquilômetros de Brasília, ____ gente disputando os restos com os animais.(Fonte: Revista VEJA, edição 1735)a) à, a, a, há, háb) a, à, à há, ac) a, a, a, a, hád) à, a, a, à, háe) a, à, à, há, a

Page 66: 1500 Port & Mat

Gabarito: C

(Fiscal do Pará/2002) Assinale a opção que preenche as lacunas de forma gramaticalmente correta.No que diz respeito ____ taxa de inflação, ainda que os resultados estejam longe da meta (mais de 7% ante____meta de 4%), é preciso reconhecer que diante dos acontecimentos de 2001 não se trata de um mauresultado. Todossabemos que os “choques de oferta” não se prestam ____ ser controlados facilmente pelamanipulação da taxa de juros e que freqüentemente, quando ocorre um choque é melhor encontrar umcaminho mais longo para retornar ____ meta do que forçar uma volta rápida com maiores custos em matériade crescimento.(Antonio Delfim Netto)a) à – a – a – àb) a – à – à – a34c) à – a – à – ad) a – a – a – ae) a – a – à – a

Gabarito: A

(AFC STN/2002) Assinale a opção que preenche corretamente as lacunas do texto abaixo.A substituição da conta Movimento do Governo no Banco do Brasil pela Conta Única do Tesouro noBanco Central, em 1988, contribuiu para que ___ administração e o controle das finanças federaisestivessem associados ____ execução financeira das unidades gestoras. A implantação da Conta Únicaeliminou mais de cinco mil contas bancárias governamentais, permitindo o controle mais eficaz do fluxo decaixa do Governo. Paralelamente, ocorreu ____ unificação dos orçamentos, eliminando-se o orçamentomonetário e, por conseguinte, atrelando os gastos governamentais _____ prévia autorização do CongressoNacional. Ainda ao final dos anos 80, o Tesouro Nacional assume as atividades relativas aos Programas deFomento ____ Agricultura e ____ Exportações, transferidos do Banco Central, assim como das atividadesrelativas ao planejamento e administração da Dívida Mobiliária Interna.(Trecho adaptado de http://www.stn.fazenda.gov.br)a) à / à / a / à / a / asb) a / à / a / a / à / àsc) a / a / à / à / à / ad) à / a / a / a / a / àe) a / a / à / a / à / as

Gabarito: B

(TCPR 2002/2003)Monteiro Lobato, ao afirmar que "um país se faz com homens e livros", por certo indicou o caminho daspedras àqueles que, descuidadamente, promovem a história sem a preocupação de seu registro e que, porconseqüência, legam ao pó do esquecimento tudo o que foi feito – certo ou errado – ou deixado de fazer. Oshomens fazem a história. Os livros registram a história. Sem estes, os exemplos do passado, os

Page 67: 1500 Port & Mat

conhecimentos técnicos e científicos armazenados, o testemunho e as provas colhidas não seriam repassadosàs gerações futuras, o que comprometeria a chamadaevolução.www.tcparaná.gov.brJulgue a assertiva abaixo.e) O emprego do sinal indicativo de crase em “àqueles”(l.2) é exigido pela regência do verbo “indicou”.(TCPR 2002/2003) Assinale a opção gramaticalmente incorreta.a) As Entidades Fiscalizadoras Superiores têm-se destacado em diversos países como órgãos fundamentaispara a consolidação de Estados democráticos e para a busca de implantação de bons governos.b) A atuação das Entidades Fiscalizadoras tem contribuído decisivamente para aumentar a transparência nosetor público, para garantir a transparência das ações governamentais e para melhorar a gestão pública deforma geral. As instituições de controle são essenciais para garantir que mudanças no setor público ereformas administrativas visem sempre ao benefício da sociedade.c) São os auditores governamentais que podem avaliar se agências públicas estão atuando com eficiência, sea qualidade dos serviços públicos está sendo melhorada, se as tarefas estão sendo racionalmente divididasentre os funcionários e se os programas estão atingindo seus objetivos.d) O controle do desempenho governamental foi desenvolvido como complemento à auditoria tradicional,assim como a nova gestão pública adicionou eficácia e efetividade administrativa aos valores burocráticostradicionais de prudência e precisão processual.e) Em relação às duas principais vertentes atuais do controle, à avaliação de desempenho e o controle deconformidade, não é viável que a primeira venha a substituir integralmente a segunda.

(AFRF 2005)Enquanto o patrimônio tradicional continua sendo responsabilidade dos Estados, a promoção da culturamoderna é cada vez mais tarefa de empresas e órgãos privados. Dessa diferença derivam dois estilos de açãocultural. Enquanto os governos pensam sua política em termos de proteção e preservação do patrimôniohistórico, as iniciativas inovadoras ficam nas mãos da sociedade civil, especialmente daqueles que dispõemde poder econômico para financiar arriscando. Uns e outros buscam na arte dois tipos de ganho simbólico: osEstados, legitimidade e consenso ao aparecer como representantes da história nacional; as empresas, obterlucro e construir através da cultura de ponta, renovadora, uma imagem “não interessada” de sua expansãoeconômica.(Nestor Garcia Canclini, Culturas Híbridas, p. 33, com adaptações)

14 - Assinale a alteração na pontuação que provoca incoerência textual ou erro gramatical no texto.a) A substituição do ponto final depois de “cultural” (l.4) por doispontos.b) A substituição dos dois-pontos depois de “simbólico” (l.9) pelo sinal de ponto-e-vírgula.c) A substituição do sinal de ponto-e-vírgula depois de “nacional” (l.10) pela conjunção e.d) A inserção de uma vírgula depois de “construir” (l.11).e) A retirada da vírgula depois de “ponta” (l.11).

Gabarito: E

(Agente Tributário MS/2001) Marque a palavra, a seqüência ou o sinal de pontuação sublinhado, que foimal empregado.

Page 68: 1500 Port & Mat

O desatendimento(A) das normas regulamentares,(B) enseja a cobrança imediata do imposto, atualizadomonetariamente e acrescido de multa e dos juros incidentes(C), desde a data da remessa da mercadoria oubem(D), inclusive no caso de venda no mercado interno da mercadoria destinada à(E) exportação.a) Ab) Bc) Cd) De) E

Gabarito: B

(Fiscal do Trabalho/2003)A sociedade baseada na liberdade contratual será sempre, em grande parte, uma sociedade de classes, cujaestrutura é defendida em vantagem dos ricos. Cumpre associar o indivíduo no processo de autoridade, isto é,o trabalhador no poder industrial. A exclusão de alguém de uma parcela do poder é, forçosamente, a exclusãodaquele dos benefícios deste. Todos deviam e devem, portanto, ter direito a uma parte dos resultados da vidasocial. E as diferenças devem existir somente quando necessárias ao bem comum. Impõe-se, pois, umaigualdade econômica maior, porque os benefícios que um homem pode obter do processo social estãoaproximadamente em função de seu poder de consumo, o que resulta do seu poder de propriedade.Assim os privilégios econômicos são contrários à verdadeira sociedade democrática. O próprio conceito deliberdade redefine-se através dos séculos, de acordo com as circunstâncias históricas e o desenvolvimentodas forças econômicas. E a liberdade, no mundo atual, só existirá de fato quando assentada na segurança eem função da igualdade. É que a verdadeira democracia, já o disse Turner, “é o direito do indivíduo decompartilhar as decisões que respeitam a sua vida e da ação necessária à execução de tais decisões”. Paraque a liberdade realmente exista, é preciso que a sociedade se estruture sobre cooperação e não sobre aexploração. E assim os homens serão livres.

(João Mangabeira, Oração do Paraninfo, proferida em Salvador, BA, em 8/12/1944, com adaptações)

Analise as seguintes afirmações a respeito do uso dos sinais de pontuação no texto.I. O emprego da vírgula depois de “classes”(l.2) é opcional e, por isso, sua retirada não causa prejuízogramatical ao texto.II. Devido ao valor explicativo do período iniciado por “A exclusão”(l.4), as regras gramaticais permitemtrocar o ponto final que o antecede pelo sinal de dois pontos, desde que se empregue o artigo com letraminúscula.III. Apesar de não ser obrigatório o emprego da vírgula depois de “Assim”(l.12), o valor conclusivo doadvérbio recomenda que aí seja inserida.IV. Por se tratar de uma citação, as regras gramaticais admitem que o período entre aspas (l.18 a 20) sejaprecedido do sinal de dois pontos,em lugar de vírgula; e, nesse caso, as aspas podem ser retiradas.

a) todos os itens estão corretos.b) nenhum item está correto.c) apenas o item II está correto.d) apenas os itens II e III estão corretos.e) apenas os itens II, III e IV estão corretos.

Page 69: 1500 Port & Mat

Gabarito: E

(ACE/1998) Indique o segmento do texto seguinte que contém erro de estruturação sintática ou depontuação.a) Já aconteceu uma vez: da Mata Atlântica, que cobria a costa brasileira do Rio Grande do Sul até oCeará, só restam hoje entre 5% a 8%, na estimativa mais otimista.b) Distante dos centros mais desenvolvidos, a Floresta Amazônica permaneceu quase intocada até a trintaanos.Nas três últimas décadas, suas árvores sofreram mais baixas do que nos quatro séculos anteriores.c) Não é um caso perdido. A Amazônia ainda está sob ocupação humana das mais ralas e há regiões com adimensão de países europeus que continuam intactas.d) Ainda se pode viajar dez horas no Rio Negro, um dos maiores da Amazônia, sem cruzar com mais dequatro ou cinco barcos e sem ver movimentação nas margens, a não ser por uma dúzia de casebres solitários.e) Mas em regiões economicamente mais atraentes, lugares que já são ocupados por vilarejos e cidades, oataque à floresta é brutal.(Baseado em Tales Alvarenga, Veja - Amazônia, 24/12/1997)

(AFC SFC/2000) Numere os trechos de modo a compor um texto coeso e coerente, e assinale a seqüênciacorreta.( ) Ela teria também eliminado a inflação e os ciclos econômicos.( ) Mas será que tudo isso está de fato transformando a economia?( ) Não há dúvida de que há uma revolução em curso na forma como nos comunicamos, trabalhamos,compramos e nos divertimos.( ) Em decorrência disso, as velhas regras econômicas e as formas tradicionais de valorização das ações nãose aplicam mais.( ) Os otimistas radicais dizem que a tecnologia da informação ajuda-a a crescer mais rapidamente.(Adaptado de Negócios Exame, p.93)a) 5, 1, 3, 2, 4b) 3, 4, 2, 5, 1c) 2, 3, 4, 1, 5d) 1, 5, 3, 4, 2e) 4, 2, 1, 5, 3

(AFC STN/2000)Numere os trechos, observando a ordem em que devem aparecer para constituírem um texto coeso ecoerente, e assinale a resposta correta.( ) Esse processo constituiu-se, então, em duas fases: 1ª) a ruptura da homogeneidade da "aristocraciaagrária"; 2ª) o aparecimento de novos tipos de agentes econômicos, sob a pressão da divisão do trabalho emescala local, regional ou nacional.( ) Ela se constitui lentamente, por vezes sob convulsões profundas, numa trajetória de ziguezagues.( ) Uma Nação não aparece e se completa de uma hora para outra.( ) Isso sucedeu no Brasil, mas de maneira a converter essa transição, do ponto de vista econômico, no

Page 70: 1500 Port & Mat

período de consolidação do capitalismo.(Florestan Fernandes, A Revolução Burguesa no Brasil, pág. 1518,com adaptações)a) 4, 2, 1, 3b) 2, 4, 3, 1c) 3, 1, 4, 2d) 1, 3, 2, 4e) 3, 2, 1, 4

1. Substitua cada conjunto destacado por uma única palavra, formada por prefixação.a) O juiz (lerá novamente) os documentos do processo.b) É necessário (fazer outra vez) todos os cálculos.c) Depois de vários anos, vou (tornar a ver) meus pais.d) Não havia motivo para pôr os interesses individuais (antes dos) interesses coletivos.e) Não há como (dizer o contrário do) que eu afirmei.f) Deixou a todos (sem proteção).g) Seu comportamento (despido de honestidade) foi punido.h) Queria uma liberdade (sem restrições).i) Os documentos foram (datados com antecedência).j) Depois de (passar além) destes limites, descansaremos.

2. Este exercício é igual ao anterior.a) Nem todos os países conseguem competir no mercado (de todas as nações.)b) Foi construída uma passagem (debaixo da terra) para evitar atropelamentos.c) (Passe uma linha por baixo) das palavras cujo significado você desconhece.d) Descobriram restos de homens (que viveram antes do período histórico) no Piauí.e) Há rastros de animais (que viveram antes do Dilúvio) naquela região.f) As civilizações (que existiam antes da chegada de Cristóvão Colombo) deixaram marcas na vida daAmérica do Sul.g) Precisava tomar injeções (dentro do músculo).

3.Explique o significado das seguintes palavras:a) reencontro, desencontrob) premeditar, pressentirc) importar, exportard) imigrante, emigrantee) imergir, emergir, submergirf) intersecçãog) imoral, amoralh) circunlóquio, colóquioi) cisandino, cisalpino, transandino, transalpinoj) co-gestãol) digressão, regressão, progressão

Page 71: 1500 Port & Mat

1 | (Fuvest/2006) A televisão tem de ser vista ...... um prisma crítico, principalmente as telenovelas, .....audiência é significativa. Temos de procurar saber ..... elas prendem tanto os telespectadores. Preenchem demodo correto as lacunas acima, respectivamente,

a) a nível de/ as quais a/ por que.

b) sobre/ que/ porquê.

c) sob/ cuja/ por que.

d) em nível de/ cuja a/ porque.

e) sob/ cuja a/ porque.

2 | (Fuvest/2006) Os verbos estão corretamente empregados apenas na frase:

a) No cerne de nossas heranças culturais se encontram os idiomas que as transmitem de geração emgeração e que assegurem a pluralidade das civilizações.

b) Se há episódios traumáticos em nosso passado, não poderemos avançar a não ser que os encaramos.

c) Estresse e ambiente hostil são apenas alguns dos fatores que possam desencadear uma explosão defúria.

d) A exigência interdisciplinar impõe a cada especialista que transcenda sua própria especialidade e quetome consciência de seus próprios limites.

e) O que hoje talvez possa vir a tornar-se uma técnica para prorrogar a vida, sem dúvida amanhã possavir a tornar-se uma ameaça.

3 | (Fuvest/2006) "Um homem precisa viajar. Por sua conta, não por meio de histórias, imagens, livros ouTV. Precisa viajar por si, com seus olhos e pés, para entender o que é seu. Para um dia plantar as suaspróprias árvores e dar-lhes valor. Conhecer o frio para conhecer o calor. E o oposto. Sentir a distância e odesabrigo para estar bem sob o próprio teto. Um homem precisa viajar para lugares que não conhece paraquebrar essa arrogância que nos faz ver o mundo como o imaginamos, e não simplesmente como é ou podeser; que nos faz professores e doutores do que não vimos, quando deveríamos ser alunos, e simplesmente irver." Amyr Klink, Mar sem fim.

Na frase “que nos faz professores e doutores do que não vimos”, o pronome sublinhado retoma a expressãoantecedente

a) “para lugares”.

b) “o mundo”.

c) “um homem”.

d) “essa arrogância”.

e) “como o imaginamos”.

4 | (Fuvest/2006) "Um homem precisa viajar. Por sua conta, não por meio de histórias, imagens, livros ouTV. Precisa viajar por si, com seus olhos e pés, para entender o que é seu. Para um dia plantar as suaspróprias árvores e dar-lhes valor. Conhecer o frio para conhecer o calor. E o oposto. Sentir a distância e odesabrigo para estar bem sob o próprio teto. Um homem precisa viajar para lugares que não conhece para

Page 72: 1500 Port & Mat

quebrar essa arrogância que nos faz ver o mundo como o imaginamos, e não simplesmente como é ou podeser; que nos faz professores e doutores do que não vimos, quando deveríamos ser alunos, e simplesmente irver." Amyr Klink, Mar sem fim.

A repetição de “precisa viajar” acentua, no contexto, o valor daquelas experiências que

a) se traduzem na exploração de nossa plena capacidade imaginativa.

b) concretizam o aprendizado das diferenças que formam a identidade pessoal.

c) ratificam a convicção de quem julga conhecer o que apenas imaginou.

d) acabam comprovando a importância de se viver tudo o que se planejou.

e) reforçam a simplicidade do prazer de um cotidiano sem surpresas.

5 | (Fuvest/2006) "o Kramer apaixonou-se por uma corista que se chamava Olga. por algum motivo nuncaconseguiam encontrar-se. ele gritava passando pela casa de Olga, manhãzinha (ela dormia): Olga, Olga,hoje estou de folga! mas nunca se viam e penso que ele sabia que se efetivamente se deitasse com ela o sonhoterminaria. sábio Kramer. nunca mais o vi. há sonhos que devem permanecer nas gavetas, nos cofres,trancados até o nosso fim. e por isso passíveis de serem sonhados a vida inteira." Hilda Hilst, Estar sendo.Ter sido. No trecho “há sonhos que devem permanecer nas gavetas, nos cofres, trancados até o nosso fim.”,o recurso de estilo que NÃO ocorre é a

a) redundância.

b) inversão.

c) gradação.

d) metáfora.

e) enumeração.

1 | (PUCRS/2004) Um escritor sempre poderá encontrar uma forma criativa para expressar algo, semincorrer em fórmulas desgastadas pelo uso excessivo. Tal, no entanto, NÃO ocorre em

a) “Na rua, sustento o caule de uma grande rosa negra, que se abre sobre mim na chuva.” (guarda-chuva)

b) “Aquela andorinha que vai sumindo cada vez mais longe – será mesmo uma andorinha? Ou minhasaudade que te mando?” (saudades)

c) “Você disse que bala mata / bala não mata ninguém / a bala que mais mata / são os olhos de meubem.” (quadrinha)

d) “Boião de leite que a noite leva com as mãos de trava para não sei quem beber e que embora levadodevagarinho vai derramando pingos brancos pelo caminho.” (lua cheia)

e) “Tu és o pão que vai matar minha fome, a água que vai saciar minha sede, o sol que aquece minhavida.” (declaração de amor)

2 | (PUCRS/2004) A repetição de expressões como “a nível de” e “de repente” ________ o texto. Também________ na categoria de cacoetes lingüísticos palavras que ________ para encerrar frases: “tá?”, “né?”,“entende?”.

Page 73: 1500 Port & Mat

a) empobrece se enquadram são utilizadas

b) empobrecem se enquadram são utilizadas

c) empobrece se enquadra utiliza-se

d) empobrecem se enquadra se utilizam

e) empobrece se enquadra utilizam-se

3 | (PUCRS/2004) INSTRUÇÃO: Responder às questões de 8 a 10 com base no texto 2.

TEXTO 2

01 Aquele fogão velho, hoje extinto – bota mais02 lenha, tem feijão cozinhando –, era muito importante03 na minha casa. Mas ele subsiste, porque as palavras04 são capazes de atiçar-lhe o fogo. Outro fogo.05 Imersa no tempo, a casa também se foi. Mas ela06 não desapareceu. Ainda existem as árvores, aquela07 taipa velha e bonita, a fonte no canto superior do08 parreiral. Essas coisas ficaram lá, nos arredores de09 1950. Porém eu as permaneço através das palavras.

PAN, Alcides. A expressão oral e escrita. Porto Alegre: Audiovisão Prod., 1980

A frase que sintetiza a idéia central do texto é:

a) As recordações da infância são as que mais marcam o ser humano.

b) O homem perpetua o transitório mediante a linguagem.

c) Recordar é viver, repetem com freqüência os saudosistas.

d) Tudo passa, menos as recordações da infância.

e) A palavra é o elo que une o passado, o presente e o futuro.

4 | (PUCRS/2004) Todas as afirmativas estão corretas, com EXCEÇÃO de:

a) A frase “Imersa no tempo, a casa também se foi.” (linha 05) pode ser assim reescrita, sem prejudicar osentido e a correção: “A casa, imersa no tempo, também se foi.” e “A casa também se foi, imersa no tempo”.

b) Os travessões das linhas 01 e 02 assinalam a intromissão de uma outra voz que não a do narrador.

c) A passagem “...porque as palavras são capazes de atiçar-lhe o fogo.” (linhas 03 e 04) está empregadaem sentido figurado.

d) O pronome “lhe” tem o mesmo sentido e a mesma função em “capazes de atiçar-lhe o fogo.” (linha 04)e em “A infância traz-lhe tristes lembranças”.

e) O pronome “as” (linha 09) refere-se a “coisas” (linha 08) que, por sua vez, refere-se a “fogão” (linha01), “casa” (linha 03), “árvores” (linha 06), “taipa” (linha 07) e “fonte” (linha 07).

5 | (PUCRS/2004) No texto, o autor vale-se de conjunções para estabelecer uma relação de ________ entre

Page 74: 1500 Port & Mat

________ e ________.

a) oposição efemeridade permanência

b) comparação o antigo o moderno

c) anterioridade passado presente

d) conseqüência infância idade adulta

e) tempo fantasia memória

1 | (FGV-SP 2005) Assinale a alternativa que não é abonada pela norma culta, quanto à regência.

a) Tratou-o com fidalguia, como a um padre.

b) Não lhe perguntou nada, apenas concordou com o que ele dizia.

c) É claro que Jesus a ama!

d) José agradeceu o homem que lhe trouxera o presente e retirou-se.

e) O chefe não lhe permitiu atender o cliente.

2 | (FGV-SP 2005) Assinale a alternativa em que um verbo, tomando outro sentido, tem alterada a suapredicação.

a) O alfaiate virou e desvirou o terno, à procura de um defeito. / Francisco virou a cabeça para o lado,indiferente.

b) Clotilde anda rápido como um raio. / Clotilde anda adoentada ultimamente.

c) A mim não me negam lugar na fila. / Neguei o acesso ao prédio, como me cabia fazer.

d) Não assiste ao prefeito o direito de julgar essa questão. / Não assisti ao filme que você mencionou.

e) Visei o alvo e atirei. / As autoridades portuárias visaram o passaporte.

3 | (FGV-SP 2005) Assinale a alternativa em que a ausência da preposição, antes do pronome relativo que,está de acordo com a norma culta.

a) É uma quantia vultosa, que o Estado não dispõe: falta-lhe numerário.

b) Vi claramente o bolso que você pôs o dinheiro nele.

c) Não interessava perguntar qual a agência que o remetente enviou a carta.

d) A garota que eu gosto não está namorando mais. Chegou a minha oportunidade.

e) Essa era a declaração que o alcaide insistia em fazer.

4 | (FGV-SP 2005) Assinale a alternativa em que a grafia das palavras está correta.

a) Beneficiente, asterístico, Ciclano, sombrancelha, excessão.

b) Estorno, beneficente, pretensão, Sicrano, assessor.

c) Auto-falante, eletrecista, asterístico, exceção, losângulo.

d) Estorno, previlégio, prazeiroso, sombrancelha, pretenção.

Page 75: 1500 Port & Mat

e) Estorno, privilégio, beneficiente, acessor, celebral.

5 | (FGV-SP 2005) Assinale a alternativa em que as formas mal ou mau estão utilizadas de acordo com anorma culta.

a) Mau-agradecidas, as juízas se postaram diante do procurador, a exigir recompensas.

b) Seu mal humor ultrapassava os limites do suportável.

c) Mal chegou a dizer isso, e tomou um sopapo que o lançou longe.

d) As respostas estavam mau dispostas sobre a mesa, de forma que ninguém sabia a seqüência correta.

e) Então, mau ajeitada, desceu triste para o salão, sem perceber que alguém a observava.

1 | (Cásper Líbero/2003) Assinale a alternativa que apresenta desvio em relação à norma culta:

a) Teve o desejo vago de realizar qualquer ação notável que espantasse o irmão e lhe despertasseatenção.

b) As abas do chapéu jogado para trás, preso debaixo do queixo pela correia, aumentavam-lhe o rostoqueimado, faziam-lhe um círculo enorme em torno da cabeça.

c) Naquele momento Fabiano lhe causava grande admiração.

d) A idéia surgiu-lhe na tarde em que Fabiano botou os arreios na égua alazã e entrou a amansar-lhe.

e) Ao passar pelo jatobá, virou-lhe o rosto.

2 | (Cásper Líbero/2003) “Descenso ameaça varrer três décadas de históriaInter, Fla e Palmeiras, que dominaram nos anos 70, 80 e 90, lutam para não cair no Brasileiro.” (Folha de S.Paulo, 10/11/2002. Esporte. p. D 1).Aliando intuição lingüística, conhecimento do processo de formação de palavras e percepção contextual,pode-se afirmar que a palavra descenso significa:

a) transmitir algo aos próprios descendentes.

b) movimento descendente; descida, deposição.

c) falta de juízo, de responsabilidade.

d) negação de um recenseamento.

e) ato ou efeito de descentrar; desviar-se do centro.

3 | (Cásper Líbero/2003) Leia o fragmento de texto abaixo:“O livro, localizei-o pela internet num antiquário orientalístico de Leyden, na Holanda. Uma demonstraçãoassim de que as lições do império mongol acerca da centralidade das redes de comunicação foram, se bem quecom atraso, devidamente assimiladas”. (Nelson Ascher, Folha de S. Paulo, 23/11/02).Assinale a alternativa que corresponda à figura de construção (ou de sintaxe) empregada na frase “O livro,localizei-o pela internet”:

a) silepse.

b) anáfora.

Page 76: 1500 Port & Mat

c) elipse.

d) polissíndeto.

e) anacoluto.

4 | (Cásper Líbero/2003) “A hora era de muito sol - o povo caçava jeito de ficarem debaixo da sombra dasárvores de cedro.” (“Sorôco, sua mãe, sua filha”. Guimarães Rosa.)Assinale a alternativa em que aparece um expediente sintático igual àquele presente no excerto acima:

a) os outros não sabendo o que era, falavam, olhavam, gesticulavam, ao tempo que ela olhava só, orafixa, ora móvel, deixando a astúcia ao ponto de olhar às vezes para dentro de si, porque deixava cair aspálpebras.

b) ... via-se em todas faces pintado o espanto e o terror.

c) a plebe vociferava as mais afrontosas injúrias contra D. Leonor: e se chegassem a entrar no paço, elasem dúvida seria feita pedaços pelo tropel furioso.

d) estava designada a noite dum baile em casa de Rita Emília, quando os convidados recebemos aviso dasúbita doença de Francisco José de Souza.

e) a expedição foi formada por um grupo de aventureiros que costumavam viajar até os confins do sertão.

5 | (Cásper Líbero/2003) “Ele, por detrás de si mesmo, pondo-se de parte, em ambíguos âmbitos e momentos,como se a vida fosse ocultável; não o conheceriam através de figuras.” (“Nada e a nossa condição”.Guimarães Rosa)Assinale a opção em que a palavra “mesmo” apresenta igual valor semântico ao retratado acima:

a) a mesma testemunha foi chamada ao tribunal.

b) foi sempre o mesmo na defesa das minorias.

c) viajar ou não era o mesmo para ele.

d) mesmo as pessoas que se diziam amigas, agora lhe viram o rosto.

e) a duquesa mesma veio nos receber.

1 | (Fuvest/2001) - A única frase que NÃO apresenta desvio em relação à regência (nominal e verbal)recomendada pela norma culta é:

a) O governador insistia em afirmar que o assunto principal seria “as grandes questões nacionais”, como que discordavam líderes pefelistas.

b) Enquanto Cuba monopolizava as atenções de um clube, do qual nem sequer pediu para integrar, asituação dos outros países passou despercebida.

c) Em busca da realização pessoal, profissionais escolhem a dedo aonde trabalhar, priorizando àempresas com atuação social.

d) Uma família de sem-teto descobriu um sofá deixado por um morador não muito consciente com alimpeza da cidade.

e) O roteiro do filme oferece uma versão de como conseguimos um dia preferir a estrada à casa, a paixãoe o sonho à regra, a aventura à repetição.

Page 77: 1500 Port & Mat

2| (Fuvest/2001) - Considerando-se a relação lógica existente entre os dois segmentos dos provérbios adiantecitados, o espaço pontilhado NÃO poderá ser corretamente preenchido pela conjunção mas, apenas em:

a) Morre o homem, (...) fica a fama.

b) Reino com novo rei, (...) povo com nova lei.

c) Por fora bela viola, (...) por dentro pão bolorento.

d) Amigos, amigos! (...) negócios à parte.

e) A palavra é de prata, (...) o silêncio é de ouro.

3 | (Fuvest/2001) - A frase em que os vocábulos sublinhados pertencem à mesma classe gramatical, exercema mesma função sintática e têm significado diferente é:

a) Curta o curta: aproveite o feriado para assistir ao festival de curta-metragem.

b) O novo novo: será que tudo já não foi feito antes?

c) O carro popular a 12.000 reais está longe de ser popular.

d) É trágico verificar que, na televisão brasileira, só o trágico é que faz sucesso.

e) O Brasil será um grande parceiro e não apenas um parceiro grande.

4 | (Fuvest/2001) - A única frase em que as formas verbais estão corretamente empregadas é:

a) Especialistas temem que órgãos de outras espécies podem transmitir vírus perigosos.

b) Além disso, mesmo que for adotado algum tipo de ajuste fiscal imediato, o Brasil ainda estará muitolonge de tornar-se um participante ativo do jogo mundial.

c) O primeiro-ministro e o presidente devem ser do mesmo partido, embora nenhum fará a sociedade emque eu acredito.

d) A inteligência é como um tigre solto pela casa e só não causará problema se o suprir de carne e omanter na jaula.

e) O nome secreto de Deus era o princípio ativo da criação, mas dizê-lo por completo equivalia a umsacrilégio, ao pecado de saber mais do que nos convinha.

5 | (Fuvest/2001) - A única frase que NÃO apresenta desvio em relação à concordância verbal recomendadapela norma culta é:

a) A lista brasileira de sítios arqueológicos, uma vez aceita pela Unesco, aumenta as chances depreservação e sustentação por meio do ecoturismo.

b) Nenhum dos parlamentares que vinham defendendo o colega nos últimos dias inscreveram-se parafalar durante os trabalhos de ontem.

c) Segundo a assessoria, o problema do atraso foi resolvido em pouco mais de uma hora, e quem fariaconexão para outros Estados foram alojados em hotéis de Campinas.

d) Eles aprendem a andar com a bengala longa, o equipamento que os auxilia a ir e vir de onde estiverpara onde entender.

e) Mas foram nas montagens do Kirov que ele conquistou fama, especialmente na cena "Reino das

Page 78: 1500 Port & Mat

Sombras", o ponto alto desse trabalho.

1 | (Fuvest/2000) - A explosão dos computadores pessoais, as “infovias”,as grandes redes - a Internet eWorld Wide Web -atropelaram o mundo. Tornaram as leis antiquadas, reformularam a economia,reordenaram prioridades, redefiniram os locais de trabalho, desafiaram constituições, mudaram o conceito derealidade e obrigaram as pessoas a ficar sentadas, durante longos períodos de tempo, diante de telas decomputadores, enquanto o CD-Rom trabalha. Não há dúvida de que vivemos a revolução da informação e,diz o professor do MIT, Nicholas Negroponte, revoluções não são sutis. (Jornal do Brasil, 13/02/96)

No texto, a expressão que sintetiza os efeitos da revolução operada pela informática é

a) “atropelaram o mundo”.

b) “tornaram as leis antiquadas”.

c) “reformularam a economia”.

d) “redefiniram os locais de trabalho”.

e) “desafiaram constituições”.

2 | (Fuvest/2000) - Na planície avermelhada, os juazeiros alargavam duas manchas verdes. Os infelizestinham caminhado o dia inteiro, estavam cansados e famintos. Ordinariamente andavam pouco, mas comohaviam repousado bastante na areia do rio seco, a viagem progredira bem três léguas. Fazia horas queprocuravam uma sombra. A folhagem dos juazeiros apareceu longe, através dos galhos pelados da caatingarala. (Graciliano Ramos, Vidas secas)

Reestruturando-se o terceiro período do texto, mantém-se o sentido original apenas em:

a) A viagem progredira bem três léguas, uma vez que haviam repousado bastante na areia do rio seco,dado que ordinariamente andavam pouco.

b) Haviam repousado bastante na areia do rio seco; a viagem progredira bem três léguas porqueordinariamente andavam pouco.

c) Porque haviam repousado bastante na areia do rio seco, ordinariamente andavam pouco, e a viagemprogredira bem três léguas.

d) Ainda que ordinariamente andassem pouco, a viagem progredira bem três léguas, pois haviamrepousado bastante na areia do rio seco.

e) Em virtude de andarem ordinariamente pouco e de haverem repousado bastante na areia do rio seco, aviagem progredira bem três léguas.

1. Complete as lacunas das frases com a forma verbal indicada entre parênteses:

Page 79: 1500 Port & Mat

a) Se efetivamente ( ), serias mais insistente. (necessitar, pretérito imperfeito dosubjuntivo)b) Seu pai não ( ) às reuniões com freqüência. (comparecer, pretérito imperfeito doindicativo)c) O diretor não nos ( ) ontem. (auxiliar, pretérito perfeito do indicativo)d) Você sempre ( ) às oito horas? (chegar, presente do indicativo)e) Quem ( ) esta rotina tão tranqüila? (alterar, futuro do pretérito do indicativo)f) Já fazia muito tempo que eu ( ) a importância de ser solidário. (perceber, pretéritomais-que-perfeito do indicativo)g) Não te ( ) em situação delicada se me prestares ajuda? (colocar, futuro do presentedo indicativo)h) Talvez eu ( ) alguma alteração no seu ânimo. (perceber, presente do subjuntivo)i) Quando ( ) a verdade, mostrai-a a todos. (descobrir, futuro do subjuntivo)

RELATÓRIO

Jorge Miguel

Senhor Superintendente

Tendo sido designado por Vossa Senhoria para apurar as denúncias de irregularidades ocorridas noaeroporto de Marília, submeto à apreciação de Vossa Senhoria o relatório das diligências que nesse sentidoefetuei.

No dia 23 de julho de 1988 dirigi-me ao senhor Raimundo Alves Correia, encarregado do aeroportodaquela cidade, para que permitisse fosse interrogado o funcionário João Romão, acusado de ter furtado umamáquina de escrever Olivetti n. 146.801, pertencente ao patrimônio do aeroporto. O acusado relatou-nos querealmente havia levado a máquina para casa na sexta-feira - 18 de março de 1988 - apenas para executaralguma tarefa de caráter particular. Não a devolveu na segunda-feira, dia 21 de março, porque faltou aoserviço por motivo de doença. Quando retornou ao serviço dia 28 de março, devolveu a máquina. A doença doacusado está comprovada pelo atestado que segue anexo ao presente relatório; a devolução da máquina nodia 28 de março foi confirmada pelo senhor Raimundo Alves Correia.

Do exposto conclui-se que me parece infundada a acusação. Não houve vontade de subtrair a máquina,mas apenas negligência do acusado em levar para casa um bem público para executar tarefa particular. Foiirresponsável. Não cometeu qualquer ato criminoso.

Não me convence seja necessário impor-se a instauração de processo administrativo. O funcionário deveser repreendido pela negligência que cometeu. É o que me cumpre levar ao conhecimento de Vossa Senhoria.

Aproveito a oportunidade para apresentar-lhe protestos de minha distinta consideração.

São Paulo, 25 de julho de 1988 Cláudio da Costa

1. O relatório é um texto de tipo:

Page 80: 1500 Port & Mat

a) descritivo;

b) narrativo;

c) argumentativo;

d) poético;

e) dramático.

2. A finalidade principal do texto é:

a) orientar o superior na tomada de uma decisão;

b) documentar oficialmente um ato irregular;

c) discutir um tema polêmico;

d) fornecer dados para uma investigação;

e) indicar funcionários passíveis de punição.

3. Não consta(m) do relatório lido:

a) o cargo da autoridade a quem é dirigido;

b) o relato dos fatos ocorridos;

c) uma preocupação literária do autor;

d) as conclusões dos fatos analisados;

e) uma fórmula de cortesia final.

4. “Tendo sido designado por Vossa Senhoria...”; esta oração inicial do texto tem valor:

a) concessivo;

b) temporal;

c) conclusivo;

d) causal;

Page 81: 1500 Port & Mat

e) consecutivo.

5. O item em que se mostra a forma abreviada correta de Vossa Senhoria é:

a) V. Sria.

b) V. Sra.

c) V. S.

d) V. Senh.

e) V. S ª.

6. “...submeto à apreciação de Vossa Senhoria...”; o acento grave indicativo da crase neste segmento sedeve a que:

a) ocorre a união da preposição a com o artigo definido feminino singular;

b) a regência do verbo submeter exige o uso da preposição a;

c) há a obrigatoriedade do emprego do artigo definido feminino singular;

d) faça parte de uma locução adverbial;

e) faça parte de uma locução prepositiva.

7. O estilo burocrático se caracteriza, entre outras coisas, pelo emprego de palavras desnecessárias; noprimeiro parágrafo do texto são exemplos desse caso:

a) denúncias; ocorridas; apreciação;

b) ocorridas; apreciação; relatório;

c) apreciação; relatório; nesse sentido;

d) relatório; denúncias; ocorridas;

e) nesse sentido; ocorridas; apreciação.

8. As datas presentes no texto têm a finalidade textual de:

Page 82: 1500 Port & Mat

a) mostrar a evolução dos acontecimentos;

b) documentar os fatos citados;

c) criar a falsa impressão de verdade;

d) valorizar o trabalho do autor do relatório;

e) facilitar a leitura do relatório.

9. “... que se anexa ao presente relatório.” ; o item abaixo em que a concordância do vocábulo anexo estácorreta é:

a) Vai anexa o atestado médico;

b) Vão anexo o atestado e a foto do funcionário;

c) Estão em anexas as fotografias pedidas;

d) Está em anexo a declaração do réu;

e) Está anexo os documentos solicitados.

10. O plural do verbo e do pronome em “dirigi-me” é:

a) dirigi-nos;

b) dirigimos-nos;

c) dirigimos-me;

d) dirigis-nos;

e) dirigimo-nos.

11. “...me parece infundada a acusação.”; o adjetivo sublinhado corresponde semanticamente a:

a) sem fundos;

b) sem fundações;

c) sem fundamento;

d) sem finalidade;

Page 83: 1500 Port & Mat

e) sem fingimento.

12. “Não cometeu qualquer ato criminoso.”; este segmento do texto corresponde a uma:

a) conclusão à que chegou o autor do relatório;

b) alegação do acusado de roubo;

c) opinião do chefe do funcionário acusado;

d) opinião do superintendente;

e) decisão do juiz encarregado do caso.

13. “...seja necessário impor-se a instauração de processo administrativo.”; o verbo sublinhado, em formareduzida, corresponde, em forma desenvolvida, a:

a) que seja imposta;

b) que se impusesse;

c) que se imponham;

d) que se impunha;

e) que se impusessem.

14. “É o que me cumpre levar ao conhecimento...”; o verbo sublinhado tem por sinônimo correto:

a) compreende;

b) cabe;

c) obriga;

d) capacita;

e) solicita.

15. Consideração tem por plural considerações; o vocábulo abaixo que faz o plural da mesma forma é:

a) cidadão;

Page 84: 1500 Port & Mat

b) escrivão;

c) irmão;

d) chapelão;

e) ademão.

16. “...para apresentar-lhe protestos...”; o pronome pessoal oblíquo está MAL colocado em:

a) Quer-lhe-ia apresentar meus votos de pronto restabelecimento;

b) Não desejo cumprimentá-lo nunca mais;

c) Nunca me digam o que fazer;

d) Fi-lo porque o quis;

e) Em o fazendo, estarás despedido.

17. “... que realmente havia levado a máquina para casa...”; a forma verbal sublinhada equivale a:

a) levava;

b) levou;

c) leva;

d) levara;

e) levasse.

18. A única irregularidade existente nos fatos narrados é:

a) a ausência prolongada do funcionário;

b) a não devolução da máquina de escrever;

c) usar-se um bem público em tarefas particulares;

d) não prevenir o chefe sobre ter levado a máquina;

e) a intenção de roubar um bem público.

Page 85: 1500 Port & Mat

19. “O acusado relatou-nos que realmente havia levado a máquina para casa...”; a frase, em discurso direto,correspondente à forma sublinhada de discurso indireto é:

a) - Eu realmente levava a máquina para casa;

b) - Eu realmente levo a máquina para casa;

c) - Eu realmente levaria a máquina para casa;

d) - Eu realmente tinha levado a máquina para casa;

e) - Eu realmente levarei a máquina para casa.

20. Em “Vossa Senhoria parece preocupado com o furto da máquina de escrever” há uma figura conhecidapor:

a) metáfora;

b) silepse de gênero;

c) silepse de número;

d) silepse de pessoa;

e) catacrese.

(Fuvest/2000) - As duas manas Lousadas! Secas, escuras e gárrulas como cigarras, desde longos anos, emOliveira, eram elas as esquadrinhadoras de todas as vidas, as espalhadoras de todas as maledicências, astecedeiras de todas as intrigas. E na desditosa cidade, não existia nódoa, pecha, bule rachado, coraçãodorido, algibeira arrasada, janela entreaberta, poeira a um canto, vulto a uma esquina, bolo encomendadonas Matildes, que seus olhinhos furantes de azeviche sujo não descortinassem e que sua solta língua, entreos dentes ralos, não comentasse com malícia estridente. (Eça de Queirós, A ilustre Casa de Ramires)

No texto, o emprego de artigos definidos e a omissão de artigos indefinidos têm como efeito,respectivamente,

a) atribuir às personagens traços negativos de caráter; apontar Oliveira como cidade onde tudo acontece.

b) acentuar a exclusividade docomportamento típico das personagens; marcar a generalidade dassituações que são objeto de seus comentários.

c) definir a conduta das duas irmãs como criticável; colocá-las como responsáveis pela maioria dosacontecimentos na cidade.

Page 86: 1500 Port & Mat

d) particularizar a maneira de ser das manas Lousadas; situá-las numa cidade onde são famosas pelamaledicência.

e) associar as ações das duas irmãs; enfatizar seu livre acesso a qualquer ambiente na cidade.

(Fuvest/2000) - Você pode dar um rolê de bike, lapidar o estilo a bordo de um skate, curtir o sol tropical, levarsua gata para surfar.

Considerando-se a variedade lingüística que se pretendeu reproduzir nesta frase, é correto afirmar que aexpressão proveniente de variedade diversa é

a) “dar um rolê de bike”.

b) “lapidar o estilo”.

c) “a bordo de um skate”.

d) “curtir o sol tropical”.

e) “levar sua gata para surfar”.

(Fuvest/2000) - Sinha Vitória falou assim, mas Fabiano resmungou, franziu a testa, achando a fraseextravagante. Aves matarem bois e cabras, que lembrança! Olhou a mulher, desconfiado, julgou que elaestivesse tresvariando. (Graciliano Ramos, Vidas secas)

Uma das características do estilo e Vidas secas é o uso do discurso indireto livre, que ocorre no trecho

a) “Sinha Vitória falou assim”.

b) “Fabiano resmungou”.

c) “franziu a testa”.

d) “que lembrança”.

e) “olhou a mulher”.

Observe o modelo:

O diretor da área financeira do Banco Central não quer que se ( ) esse tipo de falcatrua.

(encobrir)

O diretor da área financeira do Banco Central não quer que se (encubra) esse tipo de falcatrua.

a) Espero que você não ( ) essas agressões. (engolir)

b) O terapeuta sugere que nós ( ) melhor. (dormir)

c) Aquele professor, rabugentíssimo, não permite nem mesmo que alguém ( ) durante a aula. (tossir)

Page 87: 1500 Port & Mat

d) Ela espera que eu não ( ) seus segredos. (descobrir)

e) Os executivos querem que os consumidores ( ) os prejuízos advindos da má administração das empresas.(cobrir)

f) O mestre-de-obras acha melhor que se ( ) a parede com algum produto impermeabilizante. (recobrir)

Reescreva cada uma das frases abaixo, substituindo a palavra destacada por umsubstantivo abstrato e fazendo todas as transformações necessárias.a) Era um sujeito tão (altivo) que nos indignava.b) Seu olhar é tão (triste) que ficamos tentados a ajudá-lo.c) Seu caráter era tão (rijo) que impressionava até mesmo seus adversários.d) Todos sentem que seu coração é (nobre).e) É um material tão (rígido) que suporta os maiores esforços.

Nas frases seguintes, substitua as expressões destacadas por substantivos coletivos.a) O (grupo de jogadores) do clube não é dos melhores.b) O (grupo de condôminos reunidos) decidiu cortar despesas.c) Devemos proteger o (conjunto de animais) e o (conjunto de vegetais) desta região.d) A empresa aérea prometeu renovar seu (conjunto de aeronaves).e) Formou-se um grupo de (médicos experientes) para estudar o caso.f) (O conjunto dos jurados) condenou-o por crime de corrupção.g) Um (grupo de músicos) alegrou a festa.h) Aonde quer que fosse, o ministro era acompanhado por um (grupo de bajuladores).i) As palmas que se ouviam provinham de um (grupo de pessoas pagas para aplaudir).j) Aonde quer que fosse, o ministro era acompanhado por um (grupo de acompanhantese auxiliares).l) O (grupo de atores) da peça é dos melhores.m) Naquela fotografia, ele aparece rodeado de um numeroso (grupo de filhos e filhas).n) A biblioteca teve seu (conjunto de obras literárias) ampliado recentemente. Tambémfoi finalmente instalado um (arquivo de jornais e revistas).o) Comprei uma (seleção de poemas e crônicas) de Carlos Drummond de Andrade.

(PUCSP) No trecho " … o homem não fala simplesmente uma língua, não a usa, como (mero) instrumentode comunicação" , o termo sublinhado é um - nota da ledora: em todos os exercício do livro, em que as

Page 88: 1500 Port & Mat

palavras são sublinhadas ou um termo é destacado, o mesmo encontra-se entre parênteses, por motivosóbvios.

- fim da nota.

a) substantivo e significa "simples".

b) advérbio e significa "genuíno ".

c) adjetivo e significa "quase".

d) advérbio e significa "estreme".

e) adjetivo e significa "puro"

(UM-SP) Assinale a alternativa em que ambos os adjetivos não se flexionam em gênero.

a) elemento motor, tratamento médico-dentário

b) esforço vão, passeio matinal

c) juiz arrogante, sentimento fraterno

d) cientista hindu, homem célebre

e) costume andaluz, manual lúdico-instrutivo

(UFF-RJ) Das frases abaixo, apenas uma apresenta adjetivo no comparativo de superioridade. Assinale-a.

a) A palmeira é a mais alta árvore deste lugar.

b) Guardei as melhores recordações daquele dia.

c) A Lua é menor do que a Terra.

d) Ele é o maior aluno de sua turma.

e) O mais alegre dentre os colegas era Ricardo.

(FMIt-MG) Dê o grau normal dos superlativos:

Page 89: 1500 Port & Mat

a) macérrimo

b) tetérrimo

c) minutíssimo

d) personalíssimo

e) ferocíssimo

(UFU-MG) Relativamente à concordância dos adjetivos compostos indicativos de cor, uma, dentre asseguintes alternativas, está errada. Qual?

a) saia amarelo-ouro

b) papel amarelo-ouro

c) caixa vermelho-sangue

d) caixa vermelha-sangue

e) caixas vermelho-sangue

(Fuvest/97 1ªFase) - A única frase inteiramente de acordo com as normas gramaticais do padrão culto é:

a) A secretária pretende evitar que novos mandados de segurança ou liminares contra o decreto sejamexpedidas.

b) o CONTRU interditou várias dependências do prédio, inclusive o Salão Azul, cujo o madeiramentodo forro foi atacado por cupins.

c) O ministro da Agricultura da Inglaterra declarou que por hora não há motivo para sacrificar osanimais.

d) A poucos dias da eleição, os candidatos enfrentam agora uma verdadeira maratona.

e) Posso vence-las, mesmo que usem drogas, pois não é isso que as tornarão invencíveis, declarou anadadora.

(Fuvest/97 1ªFase) - Conta Rubem Braga o conselho que um amigo lhe deu certa vez; "Olhe, Rubem, faça

Page 90: 1500 Port & Mat

como eu, não tope parada com a gramática."Tratando Rubem por tu e respeitando o padrão culto, o amigo deveria dizer:

a) Olhai, Rubem, faz como eu, não enfrente a gramática.

b) Olhai, Rubem, faze como eu, não te vás atemorizar com a gramática.

c) Olha, Rubem, faças como eu, cuide de seguir a gramática.

d) Olhe,Rubem, faças como eu, evita fugir à gramática.

e) Olha, Rubem, faz como eu, não desafies a gramática.

(Fuvest/97 1ªFase) - Assinale a única frase em que a ordem de colocação das palavras NÃO produzambigüidade.

a) Rossi pede ao STF processo por calúnia contra Motta

b) É só colocar as moedas, girar a manivela e ter a escova já com pasta embalada nas mãos

c) Casal procura filho seqüestrado via Internet.

d) Câmara torna crime porte ilegal de armas

e) Regressou a Brasília depois de uma cirurgia cardíaca com cerimonial de chefe de Estado.

(FUVEST-SP) "(...) No fundo o imponente castelo. No primeiro plano a íngreme ladeira que conduz aocastelo. Descendo a ladeira numa disparada louca o fogoso ginete.

Montado no ginete o apaixonado caçula do castelão inimigo de capacete prateado com plumas brancas. Eatravessada no ginete a formosa donzela desmaiada entregando ao vento os cabelos cor de carambola." (A.de Alcântara Machado, Carmela.)

"(...)Íamos, se não me engano, pela rua das Mangueiras, quando voltando-nos, vimos um carro elegante quelevavam a trote largo dois fogosos cavalos. Uma encantadora menina, sentada ao lado de uma senhoraidosa, se recostava preguiçosamente sobre o macio estofo e deixava pender pela cobertura derreada do carro amão pequena que brincava com um leque de penas escarlates." José de Alencar, Luciola.)

Nesses excertos, observa-se que a maioria dos substantivos são modificados por adjetivos ou expressõesequivalentes. Comparando os dois textos:

a) aponte em cada um deles o efeito produzido por tal recurso lingüistico;

b) justifique sua resposta.

(FEBASP) "Os homens são os melhores fregueses" os melhores encontra-se no grau:

Page 91: 1500 Port & Mat

a) comparativo de superioridade.

b) superlativo relativo de superioridade.

c) superlativo absoluto sintético.

d) superlativo absoluto analítico de superioridade.

(PUCC-SP) O (desagradável) da questão era vê-lo de (mau) humor depois da (troca)de turno.

Na frase acima, as palavras destacadas comportam-se, respectivamente, como:

a) substantivo, adjetivo, substantivo.

b) adjetivo, advérbio, verbo.

c) substantivo, adjetivo, verbo.

d) substantivo, advérbio, substantivo.

e) adjetivo, adjetivo, verbo.

(UNIMEP-SP) Em algumas gramáticas, o adjetivo vem definido como sendo "a palavra que modifica osubstantivo". Assinale a alternativa em que o adjetivo destacado contraria a definição.

a) Li um livro lindo.

b) Beber água é saudável.

c) Cerveja gelada faz mal.

d) Gente fina é outra coisa!

e) Ele parece uma pessoa simpática.

(Fuvest/97 1ªFase) - A catacrese, figura que se observa na frase: "Montou a cavalo no burro bravo", ocorre

Page 92: 1500 Port & Mat

em:

a) Os tempos mudaram, no devagar depressa do tempo.

b) Última flor do Lácio, inculta e bela, é a um tempo esplendor e sepultura.

c) Apressadamente, todos embarcaram no trem.

d) Ó mar salgado, quando do teu sal são lágrimas de Portugal.

e) Amanheceu, a luz tem cheiro.

(Fuvest/97 1ªFase) - "Navegar é preciso, viver não é preciso".

Esta frase de antigos navegadores portugueses, retomada por Fernando Pessoa, por Caetano Veloso e sabe-se lá por quantos mais citadores ou reinventores, ganha sua última versão no âmbito da Informática, em queo termo navegar adquire outro e preciso sentido.

Na nova acepção, em tempos de Internet, o lema parece mais afirmativo do que nunca. Os olhos que hojevagueiam pela tela iluminada do onitor já não precisam nem de velas, nem de versos, nem de fados: da vidasó querem o cantinho de um quarto, de onde fazem o mundo flutuar em mares de virtualidades nunca dantesnavegados.

Indique a afirmação correta em relação ao texto.

a) O efeito sonoro explorado na seqüência de "vagueiam", "velas", "versos", "vida", "virtualidades" éconhecido como rima anterior.

b) A construção "Os olhos(...) já não precisam" é exemplo de metonímia.

c) O termo "vagueiam" está empregado no sentido de "norteiam" e é exemplo de personificação.

d) Na frase "Navegar é preciso, viver não é preciso" há um pleonasmo.

e) A construção "nem de velas, nem de versos, nem de fados" apóia-se em antíteses.

(FATEC-SP) Indique a alternativa em que não é atribuída a idéia de superlativo ao adjetivo.

a) É uma idéia agradabilíssima.

b) Era um rapaz alto, alto, alto.

c) Saí de lá hipersatisfeito.

d) Almocei tremendamente bem.

e) É uma moça assustadoramente alta

(FEI-SP) Siga o modelo:

Page 93: 1500 Port & Mat

modificação da paisagem : modificação paisagística

a) água dá chuva

b) exageros da paixão

c) atitudes de criança

d) soro contra veneno de Serpente

(EEM-SP) Dê o superlativo absoluto sintético de:

a) feliz

b) livre

(EEM-SP) Faça conforme o modelo:

alma de fora : alma exterior

a) imagem do espelho

b) parede de vidro

c) imposição da lei

d) comprimento da linha

(EPM-SP) Dê os adjetivos equivalentes às expressões em destaque.

a) programa (da tarde)

b) ciclo (da vida)

c) representante (dos alunos)

(EEM-SP) Passe para o plural.

Page 94: 1500 Port & Mat

a) borboleta azul-clara

b) borboleta cor-de-laranja

(ITA-SP) Dadas as afirmações de que os adjetivos correspondentes aos substantivos:

1. enxofre 2. chumbo 3. prata

são, respectivamente,

1.sulfúreo 2.plúmbeo 3.argênteo

verificamos que está (estão) correta(s):

a) apenas a afirmação 1.

b) apenas a afirmação 2.

c) apenas a afirmação 3.

d) apenas as afirmações 1 e 2.

e) todas as afirmações.

(UnB-DF) Relacione a primeira coluna à segunda.

(1) água

(2) chuva

(3) gato

(4) marfim

(5) prata

(6) rio

(7) não consta da lista

( ) pluvial

( ) ebúrneo

Page 95: 1500 Port & Mat

( ) felino

( ) aquilino

( ) argênteo

a seqüência correta é:

a) 7, 7, 3, 1, 7.

b) 6, 3, 7, 1, 4.

c) 2, 4, 3, 7, 5.

d) 2, 4, 7, 1, 7.

(ITA-SP) Os superlativos absolutos sintéticos de comum, soberbo, fiel, miúdo são,respectivamente:

a) comuníssimo, super, fielissimo, minúsculo.

b) comuníssimo, sobérrimo, fidelíssimo, minúsculo.

c) comuníssimo, superbíssimo, fidelíssimo, minutíssimo.

d) comunérrimo, soberrimo, fidelíssimo, miudérrimo.

e) comunérrimo, sobérrimo, fielíssimo, mi-nutíssimo.

(ITA-SP) Os adjetivos lígneo, gípseo, níveo, braquial significam, respectivamente:

a) lenhoso, feito de gesso, alvo, relativo ao braço.

b) lenhoso, feito de gesso, nivelado, relativo ao crânio.

c) lenhoso, rotativo, abalizado, relativo ao crânio.

d) associado, rotativo, nivelado, relativo ao braço.

e) associado, feito de gesso, abalizado, relativo ao crânio.

(UM-SP) Aponte a alternativa incorreta quanto à correspondência entre a locução e o adjetivo.

Page 96: 1500 Port & Mat

a) glacial (de gelo); ósseo (de osso)

b) fraternal (de irmão); argênteo (de prata)

c) farináceo (de farinha); pétreo (de pedra)

d) viperino (de vespa); ocular (de olho)

e) ebúrneo (de marfim); insípida (sem sabor)

(ITA-SP) O plural de terno azul-claro, terno verde-mar é, respectivamente:

a) ternos azuis-claros, ternos verdes-mares.

b) ternos azuis-claros, ternos verde-mares.

c) ternos azul-claro, ternos verde-mar.

d) ternos azul-claros, ternos verde-mar.

e) ternos azuis-claro, ternos verde-mar.

(UFJF-MG) Marque:

a) se I e II forem verdadeiras

b) se I e III forem verdadeiras

c) se II e III forem verdadeiras

d) se todas forem verdadeiras

e) se todas forem falsas

"...eu não sou propriamente um autor defunto, mas um defunto autor... "

I. No primeiro caso, autor é substantivo; defunto é adjetivo.

II. No segundo caso, defunto é substantivo; autor é adjetivo.

III. Em ambos os casos, tem-se um substantivo composto.

Page 97: 1500 Port & Mat

(CESGRANRIO-RJ) Assinale a alternativa em que o termo cego(s) é um adjetivo.

a) "Os cegos, habitantes de um mundo esquemático, sabem aonde ir..."

b) "O cego de Ipanema representava naquele momento todas as alegorias da noite escura da alma... "

c) "Todos os cálculos do cego se desfaziam na turbulência do álcool."

d) "Naquele instante era só um pobre cego."

e)"... da Terra que é um globo cego girando no caos."

(UFSC) Observe as proposições abaixo:

01. Poucos autores escrevem poemas herói-cômicos.

02. Os cabelos castanhos-escuros emolduravam-lhe o semblante juvenil.

04. Vestidos vermelhos e amarelo-laranja foram os mais vendidos na exposição.

08. As crianças surdo-mudas foram encaminhadas à clínica para tratamento.

16. Discutiu-se muito a respeito de ciências político-sociais na última assembléia dos professores.

32. As sociedades luso-brasileira adquiriram novos livros de autores portugueses.

Marque as frases corretas e some os valores que lhes são atribuídos.

(UNIMEP-SP) O adjetivo está mal flexionado em grau em:

a) livre: libérrimo

b) magro: macérrimo

c) doce: docílimo

d) triste: tristíssimo

e) fácil: facílimo

Page 98: 1500 Port & Mat

(CEFET-PR) Siga o exemplo:

Não chame a torre de alta, mas de altíssima.

Não considero sua atitude nobre, mas ( ) .

(PUCSP) No trecho " … o homem não fala simplesmente uma língua, não a usa, como (mero) instrumentode comunicação" , o termo sublinhado é um - nota da ledora: em todos os exercício do livro, em que aspalavras são sublinhadas ou um termo é destacado, o mesmo encontra-se entre parênteses, por motivosóbvios.

- fim da nota.

a) substantivo e significa "simples".

b) advérbio e significa "genuíno ".

c) adjetivo e significa "quase".

d) advérbio e significa "estreme".

e) adjetivo e significa "puro".

(UM-SP) Assinale a alternativa em que ambos os adjetivos não se flexionam em gênero.

a) elemento motor, tratamento médico-dentário

b) esforço vão, passeio matinal

c) juiz arrogante, sentimento fraterno

d) cientista hindu, homem célebre

e) costume andaluz, manual lúdico-instrutivo

Page 99: 1500 Port & Mat

(UFF-RJ) Das frases abaixo, apenas uma apresenta adjetivo no comparativo de superioridade. Assinale-a.

a) A palmeira é a mais alta árvore deste lugar.

b) Guardei as melhores recordações daquele dia.

c) A Lua é menor do que a Terra.

d) Ele é o maior aluno de sua turma.

e) O mais alegre dentre os colegas era Ricardo.

(FMIt-MG) Dê o grau normal dos superlativos:

a) macérrimo

b) tetérrimo

c) minutíssimo

d) personalíssimo

e) ferocíssimo

(UFU-MG) Relativamente à concordância dos adjetivos compostos indicativos de cor, uma, dentre asseguintes alternativas, está errada. Qual?

a) saia amarelo-ouro

b) papel amarelo-ouro

c) caixa vermelho-sangue

d) caixa vermelha-sangue

e) caixas vermelho-sangue

Page 100: 1500 Port & Mat